You are on page 1of 56

Business Essentials, 9e (Ebert/Griffin)

Chapter 7 Operations Management and Quality


1) Firms that make only tangible products are engaged in goods production.
Answer: TRUE
Eplanation: !oods are tangible products" while ser#ices may be either tangible or intangible.
$age Re%: 1&&
'i%%iculty: Easy
(b)ecti#e: *.1
+earning (utcome: 'iscuss the %actors that in%luence decisions about organi,ational structure
-kill: .oncept
/) 0n a low1contact system" the customer must be a part o% the system to recei#e the ser#ice.
Answer: FA+-E
Eplanation: An eample o% a low1contact system is mail deli#ery" where the customer need not
be present to recei#e the ser#ice.
$age Re%: 1*2
'i%%iculty: Easy
(b)ecti#e: *./
+earning (utcome: 'iscuss the %actors that in%luence decisions about organi,ational structure
-kill: .oncept
3) A %irm4s capacity depends on both how many people it employs and the number and si,e o% its
%acilities.
Answer: TRUE
Eplanation: .apacity is the amount o% a product that a company can produce under normal
conditions.
$age Re%: 1*3
'i%%iculty: Easy
(b)ecti#e: *.5
+earning (utcome: 'iscuss the %actors that in%luence decisions about organi,ational structure
-kill: .oncept
5) 0% parts A" 6" and . must be produced this week" a detailed schedule will indicate the
se7uence o% work.
Answer: TRUE
Eplanation: 'etailed schedules show day1to1day acti#ities that will occur in production.
$age Re%: 1**
'i%%iculty: Easy
(b)ecti#e: *.8
+earning (utcome: 'iscuss the %actors that in%luence decisions about organi,ational structure
-kill: .oncept
1
.opyright 9 /213 $earson Education" 0nc. $ublishing as $rentice :all
8) ;uality impro#ement teams are groups o% employees %rom #arious work areas who de%ine"
analy,e" and sol#e common production problems.
Answer: TRUE
Eplanation: ;uality impro#ement teams seek to impro#e both their own work methods and the
products they make.
$age Re%: 1<5
'i%%iculty: Easy
(b)ecti#e: *.*
+earning (utcome: 'iscuss the %actors that in%luence decisions about organi,ational structure
-kill: .oncept
&) 0-( =222 is a certi%ication program attesting to the %act that a %actory" laboratory" or o%%ice has
impro#ed its en#ironmental per%ormance.
Answer: FA+-E
Eplanation: This is a description o% 0-( 15222.
$age Re%: 1<5
'i%%iculty: >oderate
(b)ecti#e: *.*
+earning (utcome: 'iscuss the roles o% ethics and corporate responsibility in business
-kill: .oncept
*) Each stage in the supply chain adds #alue %or the %inal customer.
Answer: TRUE
Eplanation: The supply chain starts with raw1materials suppliers and continues until the
product reaches the end customer.
$age Re%: 1<8
'i%%iculty: Easy
(b)ecti#e: *.<
+earning (utcome: 'iscuss the %actors that in%luence decisions about organi,ational structure
-kill: .oncept
<) 6y turning raw materials into %inished goods" production creates %orm utility.
Answer: TRUE
Eplanation: Form utility is created when raw materials and human skills are con#erted into
%inished goods and ser#ices.
$age Re%: 1&*
'i%%iculty: >oderate
(b)ecti#e: *./
+earning (utcome: 'iscuss the %actors that in%luence decisions about organi,ational structure
-kill: .oncept
/
.opyright 9 /213 $earson Education" 0nc. $ublishing as $rentice :all
=) ?hen a company turns out ornaments in time %or .hristmas" it creates time utility.
Answer: TRUE
Eplanation: Time utility is created when products are made a#ailable %or consumers when they
want them.
$age Re%: 1&*
'i%%iculty: >oderate
(b)ecti#e: *./
+earning (utcome: 'escribe the ma)or components o% e%%ecti#e distribution
-kill: .oncept
12) -er#ices are more tangible" more customi,ed" and more storable than most products.
Answer: FA+-E
Eplanation: -er#ices are less tangible and less storable than goods.
$age Re%: 1&<
'i%%iculty: >oderate
(b)ecti#e: *./
+earning (utcome: 'escribe the ma)or components o% e%%ecti#e distribution
-kill: .oncept
11) !etting a haircut and ha#ing your lawn mowed are both eamples o% low1contact ser#ices.
Answer: FA+-E
Eplanation: !etting a haircut is a high1contact ser#ice.
$age Re%: 1*2
'i%%iculty: >oderate
(b)ecti#e: *./
+earning (utcome: 'iscuss the %actors that in%luence decisions about organi,ational structure
-kill: .oncept
1/) A company typically selects the kind o% production used by most o% its competitors.
Answer: FA+-E
Eplanation: A company picks the one that best achie#es its larger business strategy.
$age Re%: 1*2
'i%%iculty: >oderate
(b)ecti#e: *.3
+earning (utcome: 'iscuss the %actors that in%luence decisions about organi,ational structure
-kill: .oncept
13) 0n a same1steps layout" e7uipment and people are set up to produce one type o% good in a
%ied se7uence o% steps and are arranged according to its production re7uirements.
Answer: TRUE
Eplanation: A same1steps layout allows %or the e%%icient use o% an assembly line" where
production operations are set up along a product1%low line.
$age Re%: 1*8
'i%%iculty: >oderate
(b)ecti#e: *.5
+earning (utcome: 'iscuss the %actors that in%luence decisions about organi,ational structure
-kill: .oncept
3
.opyright 9 /213 $earson Education" 0nc. $ublishing as $rentice :all
15) An assembly line is an eample o% a custom1products layout.
Answer: FA+-E
Eplanation: An assembly line is a same1steps layout.
$age Re%: 1*5
'i%%iculty: >oderate
(b)ecti#e: *.5
+earning (utcome: 'iscuss the %actors that in%luence decisions about organi,ational structure
-kill: .oncept
18) A process %lowchart identi%ies the se7uence o% production acti#ities" mo#ements o% materials"
and work per%ormed at each stage o% the process.
Answer: TRUE
Eplanation: A process %lowchart can be analy,ed to isolate waste%ul acti#ities" sources o% delay"
and other ine%%iciencies.
$age Re%: 1*&
'i%%iculty: >oderate
(b)ecti#e: *.5
+earning (utcome: 'iscuss the %actors that in%luence decisions about organi,ational structure
-kill: .oncept
1&) -ta%% schedules identi%y which products will be produced and when.
Answer: FA+-E
Eplanation: -ta%% schedules" in general" speci%y assigned working times in upcoming days@
perhaps %or as many as 32 days or more@%or each employee on each work shi%t.
$age Re%: 1**
'i%%iculty: >oderate
(b)ecti#e: *.8
+earning (utcome: 'iscuss the %actors that in%luence decisions about organi,ational structure
-kill: .oncept
1*) >aterials management in#ol#es the %low o% materials inside o% the production %acility" while
distribution management in#ol#es the %low o% %inished goods.
Answer: FA+-E
Eplanation: >aterials management is the process by which managers plan" organi,e" and
control the %low o% materials %rom design through distribution o% %inished goods.
$age Re%: 1<2
'i%%iculty: 'i%%icult
(b)ecti#e: *.&
+earning (utcome: 'iscuss the %actors that in%luence decisions about organi,ational structure
-kill: .oncept
5
.opyright 9 /213 $earson Education" 0nc. $ublishing as $rentice :all
1<) Aust1in1time BA0T) production systems normally re7uire large amounts o% Csa%ety stockC to be
maintained in a %irm4s warehouse.
Answer: FA+-E
Eplanation: A0T production reduces in#entory le#els by bringing needed materials together only
at the precise time they are needed %or production andDor shipment to the customer.
$age Re%: 1<2
'i%%iculty: >oderate
(b)ecti#e: *.&
+earning (utcome: 'iscuss the %actors that in%luence decisions about organi,ational structure
-kill: .oncept
1=) Aapanese companies adapted the concept o% 7uality circles %rom 7uality impro#ement teams
used by U.-. companies.
Answer: FA+-E
Eplanation: ;uality impro#ement teams are patterned a%ter the Aapanese concept o% 7uality
circles.
$age Re%: 1<5
'i%%iculty: >oderate
(b)ecti#e: *.*
+earning (utcome: 'iscuss the %actors that in%luence decisions about organi,ational structure
-kill: .oncept
/2) 0-( =222 is a certi%ication program attesting that a %actory" laboratory" or o%%ice has met the
rigorous re7uirements set by the 0nternational (rgani,ation %or -tandardi,ation.
Answer: TRUE
Eplanation: >ore than 1&2 countries ha#e adopted 0-( =222 as a national standard.
$age Re%: 1<5
'i%%iculty: >oderate
(b)ecti#e: *.*
+earning (utcome: 'iscuss the roles o% ethics and corporate responsibility in business
-kill: .oncept
/1) The goal o% supply chain management is better o#erall %low in the system.
Answer: TRUE
Eplanation: A%ter se#eral companies work to impro#e the %low o% materials through the system"
the ultimate reward is better #alue %or customers.
$age Re%: 1<&
'i%%iculty: >oderate
(b)ecti#e: *.<
+earning (utcome: 'iscuss the %actors that in%luence decisions about organi,ational structure
-kill: .oncept
8
.opyright 9 /213 $earson Education" 0nc. $ublishing as $rentice :all
//) -uccess%ul companies" such as 3> and FedE" use the same operations strategy.
Answer: FA+-E
Eplanation: .ompanies adopt the kind o% production that achie#es their larger business
strategy.
$age Re%: 1*1
'i%%iculty: >oderate
(b)ecti#e: *.3
+earning (utcome: 'iscuss the %actors that in%luence decisions about organi,ational structure
-kill: .oncept
/3) (perations capability is a special ability that production does especially well to outper%orm
the competition.
Answer: TRUE
Eplanation: The chosen operations capability should be compatible with the o#erall business
strategy.
$age Re%: 1*2
'i%%iculty: >oderate
(b)ecti#e: *.3
+earning (utcome: 'iscuss the %actors that in%luence decisions about organi,ational structure
-kill: .oncept
/5) E#en ecellent %irms %ocus on one competence at a time.
Answer: FA+-E
Eplanation: Ecellent %irms learn" o#er time" how to achie#e more than )ust one competence.
$age Re%: 1*/
'i%%iculty: >oderate
(b)ecti#e: *.3
+earning (utcome: 'iscuss the %actors that in%luence decisions about organi,ational structure
-kill: .oncept
/8) According to the tet" to ensure that truckloads o% merchandise %low 7uickly to stores" ?al1
>art distribution centers are located near the hundreds o% stores that they supply" not near the
companies that supply them.
Answer: TRUE
Eplanation: At ?al1>art" managers o% the company4s huge distribution centers regard ?al1
>art outlets as their customers.
$age Re%: 1*5
'i%%iculty: >oderate
(b)ecti#e: *.5
+earning (utcome: 'escribe the ma)or components o% e%%ecti#e distribution
-kill: .oncept
&
.opyright 9 /213 $earson Education" 0nc. $ublishing as $rentice :all
/&) A detailed schedule will indicate how many employees will be working on a gi#en shi%t.
Answer: FA+-E
Eplanation: A detailed schedule shows day1to1day acti#ities that will occur in production. A
sta%% schedule identi%ies how many employees will be working.
$age Re%: 1**
'i%%iculty: >oderate
(b)ecti#e: *.8
+earning (utcome: 'iscuss the %actors that in%luence decisions about organi,ational structure
-kill: .oncept
/*) (perations control includes materials management and 7uality control.
Answer: TRUE
Eplanation: 6oth acti#ities are a part o% operations control and ensure that schedules are met
and products deli#ered" both in 7uantity and in 7uality.
$age Re%: 1*=
'i%%iculty: >oderate
(b)ecti#e: *.&
+earning (utcome: 'iscuss the %actors that in%luence decisions about organi,ational structure
-kill: .oncept
/<) For manu%acturing %irms" typical materials costs make up 82 to *8 percent o% total product
costs.
Answer: TRUE
Eplanation: >aterials stakes are high %or ser#ice %irms as well.
$age Re%: 1<2
'i%%iculty: >oderate
(b)ecti#e: *.&
+earning (utcome: 'iscuss strategies %or setting and ad)usting prices
-kill: .oncept
/=) The biggest challenge o% T;> is moti#ating employees throughout the company to achie#e
7uality goals.
Answer: TRUE
Eplanation: The backbone o% T;> is moti#ating employees throughout the company to
achie#e 7uality goals. +eaders o% the 7uality mo#ement use #arious methods and resources to
%oster a 7uality %ocusE when those e%%orts succeed" employees will ultimately accept 7uality
ownership.
$age Re%: 1<3
'i%%iculty: >oderate
(b)ecti#e: *.*
+earning (utcome: 'iscuss the %actors that a%%ect moti#ation and beha#ior in the workplace
-kill: .oncept
*
.opyright 9 /213 $earson Education" 0nc. $ublishing as $rentice :all
32) >anagers o% a restaurant eat at a competing restaurant in order to identi%y desirable
impro#ements in their own operationsE this illustrates 7uality control.
Answer: FA+-E
Eplanation: .ompetiti#e product analysis is a process by which a company analy,es a
competitor4s products to identi%y desirable impro#ements.
$age Re%: 1</
'i%%iculty: >oderate
(b)ecti#e: *.*
+earning (utcome: 'escribe the skills and %unctions o% management
-kill: .oncept
31) Falue1added analysis might %ocus on both the elimination o% waste and cost minimi,ation.
Answer: TRUE
Eplanation: Falue1added analysis re%ers to the e#aluation o% all work acti#ities" material %lows"
and paperwork to determine the #alue that they add %or customersE it o%ten re#eals waste%ul or
unnecessary acti#ities that can be eliminated without )eopardi,ing customer ser#ice.
$age Re%: 1<3
'i%%iculty: >oderate
(b)ecti#e: *.*
+earning (utcome: 'escribe the skills and %unctions o% management
-kill: .oncept
3/) The sales department relies on the engineering department to send samples and 7uotes to
prospecti#e customers on a timely basisE the sales department may be considered the engineering
department4s internal customer.
Answer: TRUE
Eplanation: .ustomers are both internal and eternal.
$age Re%: 1<5
'i%%iculty: >oderate
(b)ecti#e: *.*
+earning (utcome: 'iscuss the %actors that in%luence decisions about organi,ational structure
-kill: .oncept
33) 0-( 15222 may result in a recycling program at a company.
Answer: TRUE
Eplanation: 0-( 15222 certi%ies impro#ements in en#ironmental per%ormance" etending the
0-( approach into the arena o% en#ironmental protection and ha,ardous waste management.
$age Re%: 1<5
'i%%iculty: >oderate
(b)ecti#e: *.*
+earning (utcome: 'iscuss the roles o% ethics and corporate responsibility in business
-kill: .oncept
<
.opyright 9 /213 $earson Education" 0nc. $ublishing as $rentice :all
35) A #alue chain includes an entire network o% %irms" beginning with suppliers and ending when
production is complete.
Answer: FA+-E
Eplanation: A #alue chain" also known as a supply chain" includes the %low o% in%ormation"
materials" and ser#ices that starts with raw1materials suppliers and continues adding #alue
through other stages in the operations process until the product reaches the end customer.
$age Re%: 1<8
'i%%iculty: 'i%%icult
(b)ecti#e: *.<
+earning (utcome: 'iscuss the %actors that in%luence decisions about organi,ational structure
-kill: .oncept
38) 6ecause supply chain strategy is based on the collecti#e e%%ort o% a number o% %irms" no one
%irm in the chain gains a competiti#e ad#antage.
Answer: FA+-E
Eplanation: -upply chain strategy is based on the idea that members o% the chain will gain
competiti#e ad#antage by working as a coordinated unit.
$age Re%: 1<&
'i%%iculty: >oderate
(b)ecti#e: *.<
+earning (utcome: 'iscuss the %actors that in%luence decisions about organi,ational structure
-kill: .oncept
3&) ?hich term re%ers to all the acti#ities in#ol#ed in making products@goods and ser#ices@%or
customersG
A) operations
6) processes
.) planning
') scheduling
E) per%ormance
Answer: A
Eplanation: A) C(perationsC and CproductionC re%er to the same acti#ities.
$age Re%: 1&&
'i%%iculty: Easy
(b)ecti#e: *.1
+earning (utcome: 'iscuss the %actors that in%luence decisions about organi,ational structure
-kill: .oncept
=
.opyright 9 /213 $earson Education" 0nc. $ublishing as $rentice :all
3*) ?hich o% the %ollowing is the ability o% a product to satis%y a human want or needG
A) utility
6) relati#e ad#antage
.) content
') satis%action
E) capacity
Answer: A
Eplanation: A) $roduction adds customer #alue by pro#iding utility.
$age Re%: 1&&
'i%%iculty: Easy
(b)ecti#e: *./
+earning (utcome: 'iscuss the roles o% ethics and corporate responsibility in business
-kill: .oncept
3<) ?hich type o% utility is created when a company makes products a#ailable where consumers
want themG
A) time
6) place
.) possession
') %orm
E) operations
Answer: 6
Eplanation: 6) An eample o% place utility is a theater showing a wide selection o% mo#ies in a
popular shopping mall.
$age Re%: 1&*
'i%%iculty: Easy
(b)ecti#e: *./
+earning (utcome: 'iscuss the roles o% ethics and corporate responsibility in business
-kill: .oncept
3=) ?hich term re%ers to a set o% methods and technologies used to produce a good or ser#iceG
A) contingency plan
6) production episode
.) reduction
') methods plan
E) operations process
Answer: E
Eplanation: E) 6anks use document shredding and data encryption to protect con%idential
in%ormation. 6oth are eamples o% operations processes.
$age Re%: 1&=
'i%%iculty: Easy
(b)ecti#e: *./
+earning (utcome: 'iscuss the %actors that in%luence decisions about organi,ational structure
-kill: .oncept
12
.opyright 9 /213 $earson Education" 0nc. $ublishing as $rentice :all
52) 0n which o% the %ollowing is the customer part o% the system during ser#ice deli#eryG
A) a high1contact system
6) a low1contact system
.) a customer system
') a 7uality system
E) a utility system
Answer: A
Eplanation: A) A manicure is an eample o% a high1contact system.
$age Re%: 1*2
'i%%iculty: Easy
(b)ecti#e: *./
+earning (utcome: 'iscuss the %actors that in%luence decisions about organi,ational structure
-kill: .oncept
51) ?hat term describes a special ability that production does especially well to outper%orm the
competitionG
A) supply chain management
6) total 7uality management
.) process engineering
') operations capability
E) #alue1added analysis
Answer: '
Eplanation: ') A company chooses an operation capability that is compatible with its o#erall
business strategy.
$age Re%: 1*2
'i%%iculty: Easy
(b)ecti#e: *.3
+earning (utcome: 'iscuss the %actors that in%luence decisions about organi,ational structure
-kill: .oncept
5/) ?hich o% the %ollowing is the amount o% a product that a company can produce under normal
working conditions G
A) capacity
6) output
.) e%%iciency
') burden
E) per%ormance
Answer: A
Eplanation: A) A %irm4s capacity depends on how many people it employs and the number and
si,e o% its %acilities.
$age Re%: 1*3
'i%%iculty: Easy
(b)ecti#e: *.5
+earning (utcome: 'iscuss the %actors that in%luence decisions about organi,ational structure
-kill: .oncept
11
.opyright 9 /213 $earson Education" 0nc. $ublishing as $rentice :all
53) E7uipment and people are grouped according to %unction in which type o% production layoutG
A) cellular
6) same1steps
.) custom1products
') supply1chain
E) ser#ice
Answer: .
Eplanation: .) A custom1products layout is well suited to make1to1order shops.
$age Re%: 1*5
'i%%iculty: Easy
(b)ecti#e: *.5
+earning (utcome: 'iscuss the %actors that in%luence decisions about organi,ational structure
-kill: .oncept
55) Thousands o% empty )ugs mo#e down a con#eyor belt at .olgate1$almoli#e be%ore stopping
to be %illed and mo#ing on to be labeled. ?hat type o% layout is utili,edG
A) supply1chain
6) custom1products
.) hybrid
') %ied1position
E) same1steps
Answer: E
Eplanation: E) An assembly line is a same1steps layout.
$age Re%: 1*8
'i%%iculty: Easy
(b)ecti#e: *.5
+earning (utcome: 'iscuss the %actors that in%luence decisions about organi,ational structure
-kill: Application
58) ?hich term re%ers to how well a product does what it is supposed to doG
A) capacity
6) consistency
.) reliability
') per%ormance
E) 7uality control
Answer: '
Eplanation: ') The per%ormance o% a %ast1%ood meal might be )udged by whether it tastes good
and is reasonably %illing.
$age Re%: 1*8
'i%%iculty: Easy
(b)ecti#e: *.5
+earning (utcome: 'iscuss the roles o% ethics and corporate responsibility in business
-kill: .oncept
1/
.opyright 9 /213 $earson Education" 0nc. $ublishing as $rentice :all
5&) The sameness o% product 7uality %rom unit to unit is re%erred to by which termG
A) utility
6) reliability
.) per%ormance
') consistency
E) 7uality control
Answer: '
Eplanation: ') :igh consistency at its many locations has made .ourtyard by >arriott a leader
in the lodging industry.
$age Re%: 1*8
'i%%iculty: Easy
(b)ecti#e: *.5
+earning (utcome: 'iscuss the roles o% ethics and corporate responsibility in business
-kill: .oncept
5*) ?hich o% the %ollowing is a scheduling tool that breaks down large pro)ects into steps to be
per%ormed and speci%ies the time re7uired to per%orm each oneG
A) >yers graphic
6) critical path chart
.) !antt chart
') process analysis graph
E) master schedule
Answer: .
Eplanation: .) A pro)ect manager uses a !antt chart to keep the pro)ect mo#ing on schedule.
$age Re%: 1*<
'i%%iculty: Easy
(b)ecti#e: *.8
+earning (utcome: 'iscuss the %actors that in%luence decisions about organi,ational structure
-kill: .oncept
5<) ?hich type o% system is designed %or smooth production %low to a#oid ine%%iciencies"
eliminate unnecessary in#entories" and continuously impro#e production processesG
A) 7uality system
6) lean system
.) managed system
') production system
E) %leible system
Answer: 6
Eplanation: 6) +ean production systems were pioneered by Toyota.
$age Re%: 1<2
'i%%iculty: Easy
(b)ecti#e: *.&
+earning (utcome: 'iscuss the %actors that in%luence decisions about organi,ational structure
-kill: .oncept
13
.opyright 9 /213 $earson Education" 0nc. $ublishing as $rentice :all
5=) ?hich o% the %ollowing is H(T a ma)or ob)ecti#e o% A0T productionG
A) increasing stop1and1go production
6) reducing goods in process
.) complying with schedules
') eliminating disruptions by continuous impro#ement
E) reducing in#entory le#els
Answer: A
Eplanation: A) A0T production replaces stop1and1go production with smooth mo#ement.
$age Re%: 1<2
'i%%iculty: Easy
(b)ecti#e: *.&
+earning (utcome: 'iscuss the %actors that in%luence decisions about organi,ational structure
-kill: .ritical Thinking
82) ?hich term re%ers to the ac7uisition o% the raw materials a company needs to produce its
productsG
A) de#elopment
6) purchasing
.) goods control
') 7uality control
E) wholesaling
Answer: 6
Eplanation: 6) >ost large %irms ha#e purchasing departments to buy proper ser#ices and
materials in the amounts needed.
$age Re%: 1<2
'i%%iculty: Easy
(b)ecti#e: *.&
+earning (utcome: 'iscuss the %actors that in%luence decisions about organi,ational structure
-kill: .oncept
81) ?hich o% the %ollowing re%ers to the recei#ing" storing" handling" and counting o% all raw
materials" partly %inished goods" and %inished goodsG
A) materials handling
6) in#entory control
.) 7uality control
') wholesaling
E) distribution control
Answer: 6
Eplanation: 6) 0n#entory control ensures that enough materials in#entories are a#ailable to
meet production schedules" while at the same time a#oiding epensi#e ecess in#entories.
$age Re%: 1<2
'i%%iculty: Easy
(b)ecti#e: *.&
+earning (utcome: 'iscuss the %actors that in%luence decisions about organi,ational structure
-kill: .oncept
15
.opyright 9 /213 $earson Education" 0nc. $ublishing as $rentice :all
8/) ?hich o% the %ollowing terms re%ers to all acti#ities in#ol#ed in getting 7uality products into
the marketplaceG
A) 7uality control
6) 7uality reliability
.) total 7uality management
') per%ormance 7uality
E) 7uality o#er#iew
Answer: .
Eplanation: .) T;> must consider all aspects o% a business" including customers" suppliers"
and employees.
$age Re%: 1<1
'i%%iculty: Easy
(b)ecti#e: *.*
+earning (utcome: 'escribe the skills and %unctions o% management
-kill: .oncept
83) ?hich term describes the idea that 7uality belongs to each person who creates it while
per%orming a )obG
A) 7uality control
6) 7uality ownership
.) 7uality circles
') total 7uality management
E) 7uality impro#ement teams
Answer: 6
Eplanation: 6) A goal o% total 7uality management is to moti#ate employees to accept 7uality
ownership.
$age Re%: 1</
'i%%iculty: Easy
(b)ecti#e: *.*
+earning (utcome: 'iscuss the %actors that a%%ect moti#ation and beha#ior in the workplace
-kill: .oncept
85) ?hat name is gi#en to the process by which a company analy,es another company4s product
to identi%y desirable impro#ements in its own productG
A) 7uality reliability analysis
6) benchmarking
.) per%ormance 7uality analysis
') competiti#e product analysis
E) 7uality outsourcing
Answer: '
Eplanation: ') Using competiti#e analysis" %or eample" Toshiba might take apart a Iero
copier and test each component.
$age Re%: 1</
'i%%iculty: Easy
(b)ecti#e: *.*
+earning (utcome: 'iscuss the %actors that in%luence decisions about organi,ational structure
-kill: .oncept
18
.opyright 9 /213 $earson Education" 0nc. $ublishing as $rentice :all
88) ?hich o% the %ollowing is patterned a%ter the success%ul Aapanese concept o% 7uality circlesG
A) supply chain management
6) 7uality impro#ement team
.) total 7uality management
') 7uality ownership
E) #alue1added analysis
Answer: 6
Eplanation: 6) ;uality impro#ement teams organi,e their own work" select leaders" and
address problems in the workplace.
$age Re%: 1<5
'i%%iculty: Easy
(b)ecti#e: *.*
+earning (utcome: 'iscuss the %actors that in%luence decisions about organi,ational structure
-kill: .oncept
8&) ?hich term re%ers to the %low o% in%ormation" materials" and ser#ices %rom raw1materials
suppliers through stages in the operations process until the product reaches the end customerG
A) distribution chain
6) supply chain
.) ser#ice channel
') distribution channel
E) per%ormance channel
Answer: 6
Eplanation: 6) The term supply chain re%ers to the group o% companies and stream o% acti#ities
that work together to create a product.
$age Re%: 1<8
'i%%iculty: Easy
(b)ecti#e: *.<
+earning (utcome: 'iscuss the %actors that in%luence decisions about organi,ational structure
-kill: .oncept
8*) ?hich o% the %ollowing is another term %or the supply chainG
A) the distribution chain
6) the distribution network
.) the #alue chain
') the supply network
E) the wholesale network
Answer: .
Eplanation: .) Each stage in the #alue chain adds #alue %or the %inal customer.
$age Re%: 1<8
'i%%iculty: Easy
(b)ecti#e: *.<
+earning (utcome: 'iscuss the %actors that in%luence decisions about organi,ational structure
-kill: .oncept
1&
.opyright 9 /213 $earson Education" 0nc. $ublishing as $rentice :all
8<) ?hich term re%ers to the strategy o% paying suppliers and distributors to per%orm certain
business processes or to pro#ide needed materials or ser#icesG
A) outsourcing
6) reengineering
.) controlling
') scheduling
E) warehousing
Answer: A
Eplanation: A) The decision to outsource epands supply chains.
$age Re%: 1<&
'i%%iculty: Easy
(b)ecti#e: *.<
+earning (utcome: 'iscuss the %actors that in%luence decisions about organi,ational structure
-kill: .oncept
8=) ?hen a company turns out costumes in time %or :alloween" it creates which type o% utilityG
A) ownership
6) time
.) %orm
') place
E) #alue
Answer: 6
Eplanation: 6) Time utility is created when products are made a#ailable when customers want
them.
$age Re%: 1&*
'i%%iculty: >oderate
(b)ecti#e: *./
+earning (utcome: 'iscuss the roles o% ethics and corporate responsibility in business
-kill: Application
&2) ?hen a company makes products a#ailable where they are con#enient %or consumers" it
creates which type o% utilityG
A) time
6) %orm
.) place
') possession
E) leisure
Answer: .
Eplanation: .) A %ast1%ood restaurant located in a college union is an eample o% place utility.
$age Re%: 1&*
'i%%iculty: >oderate
(b)ecti#e: *./
+earning (utcome: 'iscuss the roles o% ethics and corporate responsibility in business
-kill: .oncept
1*
.opyright 9 /213 $earson Education" 0nc. $ublishing as $rentice :all
&1) 0n a business" whose )ob is it to draw up plans to trans%orm resources into products and bring
together basic resources" such as knowledge" physical materials" e7uipment" and laborG
A) .E(
6) operations manager
.) 7uality manager
') %loor %oreman
E) supply manager
Answer: 6
Eplanation: 6) (perations managers are responsible %or ensuring that operations processes
create #alue and pro#ide bene%its to customers.
$age Re%: 1&*
'i%%iculty: >oderate
(b)ecti#e: *./
+earning (utcome: 'escribe the skills and %unctions o% management
-kill: .oncept
&/) ?hich term describes ser#ices which cannot be produced ahead o% timeG
A) trans%ormed
6) low1contact
.) unstorable
') intangible
E) consistent
Answer: .
Eplanation: .) Unstorable ser#ices cannot be used at a later time.
$age Re%: 1&=
'i%%iculty: >oderate
(b)ecti#e: *./
+earning (utcome: 'iscuss the %actors that in%luence decisions about organi,ational structure
-kill: .oncept
&3) ?hich o% the %ollowing would be considered a low1contact ser#iceG
A) surgery
6) haircut
.) massage
') electric power
E) piano lesson
Answer: '
Eplanation: ') All o% the other choices are considered high1contact ser#ices in which the
consumer must be present during the ser#ice transaction.
$age Re%: 1*2
'i%%iculty: >oderate
(b)ecti#e: *./
+earning (utcome: 'iscuss the %actors that in%luence decisions about organi,ational structure
-kill: Application
1<
.opyright 9 /213 $earson Education" 0nc. $ublishing as $rentice :all
&5) The check1processing operations at your bank would be considered to be which type o%
systemG
A) high1contact
6) low1contact
.) con#ersion
') synthetic
E) per%ormance
Answer: 6
Eplanation: 6) .heck1processing operations can be completed without the consumer being
present.
$age Re%: 1*2
'i%%iculty: >oderate
(b)ecti#e: *./
+earning (utcome: 'iscuss the %actors that in%luence decisions about organi,ational structure
-kill: Application
&8) ?hich o% the %ollowing would be considered a high1contact systemG
A) gas company
6) electric company
.) lawn care company
') barber shop
E) postal deli#ery
Answer: '
Eplanation: ') A barber4s ser#ices re7uire that the consumer be present during the ser#ice
transaction.
$age Re%: 1*2
'i%%iculty: >oderate
AA.-6: Analytic skills
(b)ecti#e: *./
+earning (utcome: 'iscuss the %actors that in%luence decisions about organi,ational structure
-kill: .ritical Thinking
&&) -a#e1A1+ot grocery stores use which type o% strategy %or attracting customersG
A) %leibility
6) 7uality
.) low1cost
') dependability
E) consistency
Answer: .
Eplanation: .) -a#e1A1+ot o%%ers items at sa#ings up to 52 percent less than con#entional %ood
chains.
$age Re%: 1*1
'i%%iculty: >oderate
(b)ecti#e: *.5
+earning (utcome: 'iscuss the %actors that in%luence decisions about organi,ational structure
-kill: Application
1=
.opyright 9 /213 $earson Education" 0nc. $ublishing as $rentice :all
&*) 3> uses which strategy %or attracting customersG
A) 7uality
6) low1cost
.) dependability
') %leibility
E) consistency
Answer: '
Eplanation: ') The %leibility strategy at 3> emphasi,es new product de#elopment.
$age Re%: 1*1
'i%%iculty: >oderate
(b)ecti#e: *.5
+earning (utcome: 'iscuss the %actors that in%luence decisions about organi,ational structure
-kill: Application
&<) >achine" woodworking" and dry cleaning shops typically use which type o% layoutG
A) product
6) custom1products
.) same1steps
') hybrid
E) make1to1stock
Answer: 6
Eplanation: 6) 0n a custom1products layout" machines and people are grouped by %unction in
the production %acilityE custom1products layouts allow %or greater %leibility and are well suited
to make1to1order shops.
$age Re%: 1*5
'i%%iculty: >oderate
(b)ecti#e: *.5
+earning (utcome: 'iscuss the %actors that in%luence decisions about organi,ational structure
-kill: Application
&=) ?hich type o% layout is designed to mo#e resources through a smooth" %ied se7uence o%
stepsG
A) same1steps
6) location
.) custom1products
') 7uality
E) make1to1order
Answer: A
Eplanation: A) A same1steps layout is set up to make one type o% product in a %ied se7uence
and is arranged according to its production re7uirements.
$age Re%: 1*8
'i%%iculty: >oderate
(b)ecti#e: *.5
+earning (utcome: 'iscuss the %actors that in%luence decisions about organi,ational structure
-kill: .oncept
/2
.opyright 9 /213 $earson Education" 0nc. $ublishing as $rentice :all
*2) Automobile" %ood1processing" and tele#ision assembly plants use which type o% layoutG
A) make1to1order
6) location
.) custom1products
') 7uality
E) same1steps
Answer: E
Eplanation: E) These production processes re7uire a %ied se7uence that is arranged according
to production re7uirements.
$age Re%: 1*8
'i%%iculty: >oderate
(b)ecti#e: *.5
+earning (utcome: 'iscuss the %actors that in%luence decisions about organi,ational structure
-kill: Application
*1) ?hich term is de%ined as the combination o% Ccharacteristics o% a product or ser#ice that bear
on its ability to satis%y stated or implied needsCG
A) 7uality
6) production
.) 7uantity
') clari%ication
E) capacity
Answer: A
Eplanation: A) -uch characteristics can include a reasonable price.
$age Re%: 1*8
'i%%iculty: >oderate
(b)ecti#e: *.5
+earning (utcome: 'iscuss the roles o% ethics and corporate responsibility in business
-kill: .oncept
*/) $er%ormance re%ers to which o% the %ollowingG
A) the principle that 7uality belongs to each person who creates it while per%orming a )ob
6) the consistency o% product 7uality %rom unit to unit
.) how well the product does what it is supposed to do
') the process by which a company analy,es a competitor4s products to identi%y desirable
impro#ement
E) pro#iding #alue by making products a#ailable when customers want them
Answer: .
Eplanation: .) $lanning %or 7uality begins when products are being designed. Early in the
process" goals are established %or both per%ormance and consistency.
$age Re%: 1*8
'i%%iculty: >oderate
(b)ecti#e: *.5
+earning (utcome: 'iscuss the roles o% ethics and corporate responsibility in business
-kill: .oncept
/1
.opyright 9 /213 $earson Education" 0nc. $ublishing as $rentice :all
*3) .onsistency re%ers to which o% the %ollowingG
A) the principle that 7uality belongs to each person who creates it while per%orming a )ob
6) the sameness o% product 7uality %rom unit to unit
.) the sum o% all acti#ities in#ol#ed in getting high17uality products into the marketplace
') the process by which a company analy,es a competitor4s products to identi%y desirable
impro#ements
E) pro#iding #alue by making products a#ailable where customers want them
Answer: 6
Eplanation: 6) This consistency is achie#ed by controlling %or consistent raw materials"
encouraging conscientious work" and maintaining e7uipment.
$age Re%: 1*8
'i%%iculty: >oderate
(b)ecti#e: *.5
+earning (utcome: 'iscuss the roles o% ethics and corporate responsibility in business
-kill: .oncept
*5) >anagers can work to reduce waste" ine%%iciency" and poor per%ormance by eamining
procedures on a step1by1step basis. ?hich term describes this processG
A) ad#ance planning
6) materials management
.) methods impro#ement
') 7uality planning
E) 7uality ownership
Answer: .
Eplanation: .) A process %lowchart is sometimes used to assist in methods impro#ement.
$age Re%: 1*&
'i%%iculty: >oderate
(b)ecti#e: *.5
+earning (utcome: 'escribe the skills and %unctions o% management
-kill: .oncept
*8) +ogan Aluminum makes coils o% aluminum that it supplies to customer companies that use it
to make be#erage cans. +ogan uses a schedule that speci%ies how many tons o% each type o% coil
will be produced each week. ?hat is this schedule calledG
A) master production schedule
6) detailed schedule
.) !antt chart
') sta%% schedule
E) $ERT chart
Answer: A
Eplanation: A) The master production schedule shows which products will be produced" and
when" in upcoming time periods.
$age Re%: 1**
'i%%iculty: >oderate
(b)ecti#e: *.8
+earning (utcome: 'escribe the skills and %unctions o% management
-kill: Application
//
.opyright 9 /213 $earson Education" 0nc. $ublishing as $rentice :all
*&) 0n operations control" production managers monitor production per%ormance by which
methodG
A) comparing results with detailed plans and schedules
6) checking on each worker on the production %loor
.) obser#ing acti#ities %rom a plat%orm abo#e the workers
') constantly talking with employees in#ol#ed
E) per%orming spot checks o% worker per%ormance
Answer: A
Eplanation: A) 0% schedules or 7uality standards aren4t met" managers can take correcti#e
action.
$age Re%: 1*=
'i%%iculty: >oderate
(b)ecti#e: *.&
+earning (utcome: 'escribe the skills and %unctions o% management
-kill: .oncept
**) ?hich o% the %ollowing is H(T one o% the areas o% materials managementG
A) transportation
6) warehousing
.) purchasing
') supplier selection
E) customer ser#ice
Answer: E
Eplanation: E) >aterials management includes supplier selection" purchasing" transportation"
warehousing" and in#entory control.
$age Re%: 1<2
'i%%iculty: >oderate
(b)ecti#e: *.&
+earning (utcome: 'escribe the skills and %unctions o% management
-kill: .oncept
/3
.opyright 9 /213 $earson Education" 0nc. $ublishing as $rentice :all
*<) ?hat is the name %or a production system in which all the needed materials and parts arri#e
at the precise moment they are re7uired %or each production stageG
A) 7uality control system
6) process control system
.) )ust1in1time production
') standardi,ed production
E) custom1products production
Answer: .
Eplanation: .) A0T production brings together all needed materials at the precise moment they
are re7uired %or each production stage" not be%ore" creating e%%icient responses to customer
orders.
$age Re%: 1<2
'i%%iculty: >oderate
(b)ecti#e: *.&
+earning (utcome: 'iscuss the %actors that in%luence decisions about organi,ational structure
-kill: .oncept
*=) ?hich o% the %ollowing is greatly reduced in a )ust1in1time production systemG
A) number o% workers needed on the line
6) number o% goods in process
.) number o% shi%ts necessary at the plant
') number o% separate operations on the assembly line
E) number o% %orepersons needed at the plant
Answer: 6
Eplanation: 6) As a result" A0T production minimi,es in#entory costs and sa#es money by
replacing stop1and1go production with smooth mo#ement.
$age Re%: 1<2
'i%%iculty: >oderate
(b)ecti#e: *.&
+earning (utcome: 'iscuss the %actors that in%luence decisions about organi,ational structure
-kill: .oncept
<2) ?hich o% the %ollowing is a component o% materials managementG
A) promotion
6) distribution
.) marketing
') warehousing
E) sales
Answer: '
Eplanation: ') >aterials management includes supplier selection" purchasing" transportation"
warehousing" and in#entory control.
$age Re%: 1<2
'i%%iculty: >oderate
(b)ecti#e: *.&
+earning (utcome: 'escribe the skills and %unctions o% management
-kill: .oncept
/5
.opyright 9 /213 $earson Education" 0nc. $ublishing as $rentice :all
<1) ?hich o% the %ollowing best describes total 7uality managementG
A) the principle that 7uality belongs to each person who creates it while per%orming a )ob
6) the consistency o% a product4s 7uality %rom unit to unit
.) the sum o% all acti#ities in#ol#ed in getting high17uality products into the marketplace
') the process by which a company analy,es a competitor4s products to identi%y desirable
impro#ements
E) a certi%ication program attesting that an operations process has met rigorous re7uirements
Answer: .
Eplanation: .) T;> begins with leadership and a desire %or continuously impro#ing both
processes and productsE it must consider all aspects o% a business" including customers" suppliers"
and employees.
$age Re%: 1<1
'i%%iculty: >oderate
(b)ecti#e: *.*
+earning (utcome: 'escribe the skills and %unctions o% management
-kill: .oncept
</) ?hich o% the %ollowing best describes 7uality ownershipG
A) an emphasis on intangible %actors in customer satis%action
6) the consistency o% a product4s 7uality %rom unit to unit
.) the sum o% all acti#ities in#ol#ed in getting high17uality products into the marketplace
') the process by which a company analy,es a competitor4s products to identi%y desirable
impro#ements
E) the principle that 7uality belongs to each person who creates it while per%orming a )ob
Answer: E
Eplanation: E) ;uality1%ocused leaders use #arious methods to %oster a 7uality %ocus among the
work%orceE when these e%%orts succeed" employees will ultimately accept 7uality ownership.
$age Re%: 1</
'i%%iculty: >oderate
(b)ecti#e: *.*
+earning (utcome: 'iscuss the %actors that a%%ect moti#ation and beha#ior in the workplace
-kill: .oncept
/8
.opyright 9 /213 $earson Education" 0nc. $ublishing as $rentice :all
<3) ?hich o% the %ollowing best describes competitive product analysisG
A) the principle that 7uality belongs to each person who creates it while per%orming a )ob
6) the consistency o% a product4s 7uality %rom unit to unit
.) the process by which a company analy,es a di%%erent company4s products to identi%y desirable
impro#ements
') the sum o% all acti#ities in#ol#ed in getting a high17uality product into the marketplace.
E) eamining a product to impro#e its %orm utility
Answer: .
Eplanation: .) This analysis helps managers decide which product %eatures are satis%actory"
which %eatures should be upgraded" and which operations processes need impro#ement" %or
eample.
$age Re%: 1</
'i%%iculty: >oderate
(b)ecti#e: *.*
+earning (utcome: 'iscuss the %actors that in%luence decisions about organi,ational structure
-kill: .oncept
<5) ?hen a worker at Toshiba takes apart a Iero copier and tests each component" it is
engaging in what acti#ityG
A) competiti#e product analysis
6) benchmarking
.) total 7uality management
') 7uality reliability analysis
E) #alue1added analysis
Answer: A
Eplanation: A) .ompetiti#e product analysis is the process by which a company analy,es a
competitor4s products to identi%y desirable impro#ements.
$age Re%: 1</
'i%%iculty: >oderate
AA.-6: Re%lecti#e thinking skills
(b)ecti#e: *.*
+earning (utcome: 'iscuss the %actors that in%luence decisions about organi,ational structure
-kill: Application
/&
.opyright 9 /213 $earson Education" 0nc. $ublishing as $rentice :all
<8) ?hich o% the %ollowing best describes #alue1added analysisG
A) a T;> tool in which groups o% employees work together to impro#e 7uality
6) the process o% e#aluating all work acti#ities" materials %lows" and paperwork to determine the
#alue that they create %or customers
.) the process by which a company implements the best practices %rom its own past per%ormance
and those o% other companies to impro#e its own products
') the process by which a company analy,es a product to identi%y possible impro#ements
E) a total company commitment to 7uality management at e#ery stage o% the process
Answer: 6
Eplanation: 6) Falue1added analysis o%ten re#eals waste%ul or unnecessary acti#ities that can be
eliminated without )eopardi,ing customer ser#ice.
$age Re%: 1<3
'i%%iculty: >oderate
(b)ecti#e: *.*
+earning (utcome: 'iscuss the %actors that in%luence decisions about organi,ational structure
-kill: .oncept
<&) Tootsie Roll 0ndustry4s corporate principle" C?e run a trim operation and continually stri#e to
eliminate waste" minimi,e costs" and implement per%ormance impro#ements"C is an eample o%
which o% the %ollowingG
A) )ust1in1time production
6) benchmarking
.) a 7uality circle
') competiti#e product analysis
E) #alue1added analysis
Answer: E
Eplanation: E) Tootsie Roll 0ndustry e#aluates all work acti#ities" material %lows" and
paperwork to determine the #alue that they add %or customers.
$age Re%: 1<3
'i%%iculty: >oderate
AA.-6: Re%lecti#e thinking skills
(b)ecti#e: *.*
+earning (utcome: 'iscuss the %actors that in%luence decisions about organi,ational structure
-kill: Application
/*
.opyright 9 /213 $earson Education" 0nc. $ublishing as $rentice :all
<*) ?hich term re%ers to collaborati#e groups o% employees %rom #arious work areas who meet
regularly to de%ine" analy,e" and sol#e common production problemsG
A) 7uality impro#ement teams
6) 7uality assurance teams
.) 7uality control teams
') T;> teams
E) 7uality re#iew teams
Answer: A
Eplanation: A) ;uality impro#ement teams organi,e their own work" select leaders" and
address problems in the workplace.
$age Re%: 1<5
'i%%iculty: >oderate
(b)ecti#e: *.*
+earning (utcome: 'iscuss the %actors that in%luence decisions about organi,ational structure
-kill: .oncept
<<) ?hich o% the %ollowing best describes 0-( =222G
A) the concept that all employees are #aluable contributors to a %irm4s business" and should be
entrusted with decisions regarding their work
6) the redesigning o% business processes to impro#e per%ormance" 7uality" and producti#ity
.) a program certi%ying that a %actory" laboratory" or o%%ice has met the 7uality management
standards o% the 0nternational (rgani,ation %or -tandardi,ation
') a certi%ication program attesting to the %act that a %actory" laboratory" or o%%ice has impro#ed
en#ironmental per%ormance
E) the process o% e#aluating all work acti#ities to determine the #alue they add %or customers
Answer: .
Eplanation: .) 0-( =222 certi%ies that a %actory" laboratory" or o%%ice has met the 7uality
management standards o% 0-(E these standards are now regarded as a national standard %or more
than 1&2 countries.
$age Re%: 1<5
'i%%iculty: >oderate
(b)ecti#e: *.*
+earning (utcome: 'iscuss the roles o% ethics and corporate responsibility in business
-kill: .oncept
/<
.opyright 9 /213 $earson Education" 0nc. $ublishing as $rentice :all
<=) ?hich program certi%ies impro#ements in en#ironmental per%ormanceG
A) 0-( =222
6) 0-( =222:/222
.) 0-( 15222
') T;> /221
E) T;> /21/
Answer: .
Eplanation: .) Etending the 0-( approach into the arena o% en#ironmental protection and
ha,ardous waste management" 0-( 15222 re7uires a %irm to de#elop an en#ironmental
management system.
$age Re%: 1<5
'i%%iculty: >oderate
(b)ecti#e: *.*
+earning (utcome: 'iscuss the roles o% ethics and corporate responsibility in business
-kill: .oncept
=2) ?hich o% the %ollowing best describes 0-( 15222G
A) the concept that all employees are #aluable contributors to a %irm4s business and should be
entrusted with decisions regarding their work
6) a certi%ication program attesting to the %act that a %actory" laboratory" or o%%ice has impro#ed
en#ironmental per%ormance
.) a certi%ication program attesting to the %act that a %actory" laboratory" or o%%ice has met the
7uality management standards o% the 0nternational (rgani,ation %or -tandardi,ation
') a standardi,ed method o% e#aluating a company4s greenhouse gas emissions
E) a certi%ication program attesting to the %act that a %actory" laboratory" or o%%ice is operating
under %ree1trade principles
Answer: 6
Eplanation: 6) 0-( 15222 re7uires a %irm to de#elop an en#ironmental management system"
documenting how the company has acted to impro#e its per%ormance in using resources and in
managing pollution.
$age Re%: 1<5
'i%%iculty: >oderate
(b)ecti#e: *.*
+earning (utcome: 'iscuss the roles o% ethics and corporate responsibility in business
-kill: .oncept
/=
.opyright 9 /213 $earson Education" 0nc. $ublishing as $rentice :all
=1) Recei#ing and storing materials" billing patients %or treatment" and %illing customer orders
%rom 0nternet sales are eamples o% which acti#ityG
A) %unction
6) techni7ue
.) process
') benchmark
E) 7uality ownership
Answer: .
Eplanation: .) E#ery business consists o% processes" which are acti#ities that are per%ormed
regularly and routinely in conducting business.
$age Re%: 1<8
'i%%iculty: >oderate
(b)ecti#e: *.*
+earning (utcome: 'iscuss the %actors that in%luence decisions about organi,ational structure
-kill: Application
=/) ?hich term re%ers to the %undamental rethinking and radical redesign o% a business acti#ity to
achie#e dramatic impro#ements in per%ormanceG
A) supply chain management
6) business process reengineering
.) total 7uality management
') 7uality ownership
E) 7uality control
Answer: 6
Eplanation: 6) Rethinking the production steps by starting %rom scratch has allowed dramatic
impro#ements in cost" 7uality" ser#ice" and speed.
$age Re%: 1<8
'i%%iculty: >oderate
(b)ecti#e: *.*
+earning (utcome: 'iscuss the %actors that in%luence decisions about organi,ational structure
-kill: .oncept
32
.opyright 9 /213 $earson Education" 0nc. $ublishing as $rentice :all
=3) ?hich o% the %ollowing best describes business process reengineeringG
A) the redesigning o% business acti#ities to impro#e per%ormance" 7uality" and producti#ity
6) a program certi%ying that a %actory" laboratory" or o%%ice has met the 7uality management
standards o% the 0nternational (rgani,ation %or -tandardi,ation
.) the principle o% looking at the supply chain as a whole in order to impro#e the o#erall %low
through the system
') a program certi%ying that a %actory" laboratory" or o%%ice has de#eloped pollution1control
standards
E) the process by which a company analy,es a di%%erent company4s products to identi%y desirable
impro#ements
Answer: A
Eplanation: A) 6usiness process reengineering %ocuses on impro#ing a business process"
rethinking each o% its steps %rom start to %inish.
$age Re%: 1<8
'i%%iculty: >oderate
(b)ecti#e: *.*
+earning (utcome: 'iscuss the %actors that in%luence decisions about organi,ational structure
-kill: .oncept
=5) ?hich o% the %ollowing best describes supply chain managementG
A) the principle o% impro#ing the supply chain by %ocusing on the slowest step in the chain
6) a limitation on the number o% suppliers allowed to supply a particular company
.) complete mo#ement o% raw materials throughout a manu%acturing or ser#ice %acility
') a cost1reduction program in which wholesalers and retailers are eliminated in an e%%ort to
entice consumers to purchase directly %rom manu%acturers
E) the principle o% looking at the supply chain as a whole in order to impro#e the o#erall %low
through the system
Answer: E
Eplanation: E) 0n addition" because customers ultimately get better #alue" supply chain
management gains competiti#e ad#antage %or each supply chain member.
$age Re%: 1<&
'i%%iculty: >oderate
(b)ecti#e: *.<
+earning (utcome: 'escribe the skills and %unctions o% management
-kill: .oncept
31
.opyright 9 /213 $earson Education" 0nc. $ublishing as $rentice :all
=8) ?hen 'ell 0nc. shares in%ormation to impro#e the o#erall %low through a system composed
o% companies working together" it is engaging in which o% the %ollowingG
A) distribution chain management
6) total 7uality management
.) supply chain management
') ser#ice channel analysis
E) supplier selection analysis
Answer: .
Eplanation: .) >ichael 'ell4s #ision in#ol#es the concept o% impro#ing per%ormance by
sharing in%ormation among supply chain membersE long1term production plans and up1to1the1
minute sales data are a#ailable to suppliers #ia the 0nternet.
$age Re%: 1<&
'i%%iculty: >oderate
(b)ecti#e: *.<
+earning (utcome: 'escribe the skills and %unctions o% management
-kill: Application
=&) ?hen Ford assembles parts into a Ford Eplorer" it creates which type o% utilityG
A) time
6) %orm
.) possession
') place
E) ownership
Answer: 6
Eplanation: 6) Form utility is created when raw materials are con#erted into %inished products.
$age Re%: 1&*
'i%%iculty: >oderate
(b)ecti#e: *./
+earning (utcome: 'iscuss the %actors that in%luence decisions about organi,ational structure
-kill: Application
=*) ?hich operations competence has Toyota gi#en greatest credit %or winning orders in the
marketplaceG
A) 7uality
6) low cost
.) %leibility
') dependability
E) eclusi#ity
Answer: A
Eplanation: A) Toyota has %ocused on creating reliable cars with an appealing %it and %inish"
and assuring that customer epectations are met or eceeded in order to build the company4s
7uality competence.
$age Re%: 1*1
'i%%iculty: >oderate
(b)ecti#e: *.3
+earning (utcome: 'iscuss the %actors that in%luence decisions about organi,ational structure
-kill: Application
3/
.opyright 9 /213 $earson Education" 0nc. $ublishing as $rentice :all
=<) ?hich operations competence has FedE gi#en greatest credit %or winning orders Bbusiness)
in the marketplaceG
A) 7uality
6) low cost
.) %leibility
') dependability
E) accessibility
Answer: '
Eplanation: ') FedE wants to assure that e#ery deli#ery is %ast and on time" as promised.
$age Re%: 1*1
'i%%iculty: >oderate
(b)ecti#e: *.3
+earning (utcome: 'iscuss the %actors that in%luence decisions about organi,ational structure
-kill: Application
==) Eamining step1by1step procedures to reduce ine%%iciency most centrally in#ol#es which
approachG
A) operations scheduling
6) per%ormance analysis
.) #alue1added analysis
') methods impro#ement
E) 7uality ownership
Answer: '
Eplanation: ') >ethods impro#ement has been used to streamline the traditional checkout
method at hotels.
$age Re%: 1*&
'i%%iculty: 'i%%icult
(b)ecti#e: *.5
+earning (utcome: 'escribe the skills and %unctions o% management
-kill: .oncept
122) ?hich scheduling tool shows the necessary se7uence o% acti#ities in a pro)ect and identi%ies
the critical pathG
A) !antt chart
6) process %lowchart
.) detailed schedule
') master production schedule
E) $ERT chart
Answer: E
Eplanation: E) The critical path is the most time1consuming set o% acti#ities in a pro)ect.
$age Re%: 1**
'i%%iculty: 'i%%icult
(b)ecti#e: *.8
+earning (utcome: 'iscuss the %actors that in%luence decisions about organi,ational structure
-kill: .oncept
33
.opyright 9 /213 $earson Education" 0nc. $ublishing as $rentice :all
121) ?hen :ewlett1$ackard simpli%ied its contracts and reduced them %rom /2 pages to as %ew
as /" it was engaging in which acti#ityG
A) #alue1added analysis
6) benchmarking
.) getting closer to the customer
') supply chain management
E) %ollow1up
Answer: A
Eplanation: A) Falue1added analysis re%ers to the e#aluation o% all work acti#ities" materials
%lows" and paperwork to determine the #alue that they add %or customers.
$age Re%: 1<3
'i%%iculty: 'i%%icult
(b)ecti#e: *.*
+earning (utcome: 'escribe the skills and %unctions o% management
-kill: Application
12/) 'e%ine the term production or operations.
Answer: The term operations Bor production) re%ers to all the acti#ities in#ol#ed in making
products@goods and ser#ices@%or customers.
Eplanation: A key di%%erence between goods and ser#ices operations is the customer4s
in#ol#ement in the latter.
$age Re%: 1&&
'i%%iculty: >oderate
(b)ecti#e: *.1
+earning (utcome: 'iscuss the %actors that in%luence decisions about organi,ational structure
-kill: .oncept
123) ?hat is an operations capabilityG
Answer: (perations capability re%ers to the acti#ity or process that production does especially
well. Each company4s operations capability matches up with its business strategy so that the
%irm4s acti#ities" %rom top to bottom" are %ocused in a particular direction.
Eplanation: For eample" the operations capability o% FedE is dependability.
$age Re%: 1*2
'i%%iculty: >oderate
(b)ecti#e: *.3
+earning (utcome: 'iscuss the %actors that in%luence decisions about organi,ational structure
-kill: .oncept
35
.opyright 9 /213 $earson Education" 0nc. $ublishing as $rentice :all
125) Eplain competiti#e product analysis.
Answer: .ompetiti#e product analysis is a process by which a company analy,es a competitor4s
products to identi%y desirable impro#ements in its own products.
Eplanation: For eample" a small business owner might study a competitor4s ?eb site to %ind
ways to impro#e her own site.
$age Re%: 1</
'i%%iculty: >oderate
(b)ecti#e: *.*
+earning (utcome: 'iscuss the %actors that in%luence decisions about organi,ational structure
-kill: .oncept
128) 'i%%erentiate between goods production and ser#ice operations.
Answer: !oods production in#ol#es tangible products" such as radios" newspapers" buses" and
tetbooks. Through ser#ice operations" %irms produce tangible and intangible ser#ice products
such as entertainment" transportation" and education.
Eplanation: !eneral Electric is a company that produces goods and pro#ides ser#ices.
$age Re%: 1&&
'i%%iculty: >oderate
(b)ecti#e: *.1
+earning (utcome: 'iscuss the %actors that in%luence decisions about organi,ational structure
-kill: .oncept
12&) 'i%%erentiate between high1 and low1contact processes. !i#e an eample o% each.
Answer: $rocesses may be classi%ied according to the etent o% customer contact as either high1
or low1contact processes. 0n a high1contact process" the customer must be a part o% the system.
Eamples o% high1contact processes include public transportation" medical ser#ices" and most
legal ser#ices. ?ith a low1contact process" customers need not be a part o% the system to recei#e
the ser#ice. Eamples include check1processing operations at the bank and lawn1care ser#ices.
Eplanation: The manager o% a high1contact process must be more concerned with the customer
eperience.
$age Re%: 1*2
'i%%iculty: >oderate
(b)ecti#e: *./
+earning (utcome: 'iscuss the %actors that in%luence decisions about organi,ational structure
-kill: .oncept
12*) ?hat is a !antt chartG
Answer: A !antt chart is a production schedule that breaks down large pro)ects into steps to be
per%ormed and speci%ies the time re7uired to per%orm each step.
Eplanation: The pro)ect manager checks the progress against the time scale and may need to
add workers i% the pro)ect %alls behind schedule.
$age Re%: 1*<
'i%%iculty: >oderate
(b)ecti#e: *.8
+earning (utcome: 'iscuss the %actors that in%luence decisions about organi,ational structure
-kill: .oncept
38
.opyright 9 /213 $earson Education" 0nc. $ublishing as $rentice :all
12<) ?hat is 7uality ownershipG
Answer: ;uality ownership is the principle o% total 7uality management that holds that 7uality
belongs to each person who creates it while per%orming a )ob.
Eplanation: A goal o% total 7uality management is moti#ating employees to accept 7uality
ownership.
$age Re%: 1</
'i%%iculty: >oderate
(b)ecti#e: *.*
+earning (utcome: 'iscuss the %actors that a%%ect moti#ation and beha#ior in the workplace
-kill: .oncept
12=) Eplain what is meant by the term utility. ?hat type o% utility is created when a pharmacy
changes its e#ening schedule to remain open %or an etra hourG
Answer: Utility is a product4s ability to satis%y a human want or need. The pharmacy creates
time utility by being open at a con#enient time %or customers.
Eplanation: The three types o% utility are %orm utility" place utility" and time utility.
$age Re%: 1&&
'i%%iculty: 'i%%icult
(b)ecti#e: *./
+earning (utcome: 'iscuss the roles o% ethics and corporate responsibility in business
-kill: Application
112) ?hy does each company4s operations capability match up with its business strategyG
Answer: Each company4s operations capability matches up with its business strategy so that the
%irm4s acti#ities " %rom top to bottom" are %ocused in a particular direction. Aligning operations
capability with business strategy helps to impro#e pro%itability.
Eplanation: For eample" FedE %ocuses on dependability which has made it a leader with
business customers.
$age Re%: 1*211*1
'i%%iculty: 'i%%icult
(b)ecti#e: *.3
+earning (utcome: 'iscuss the %actors that in%luence decisions about organi,ational structure
-kill: .oncept
111) 'escribe two alternati#es %or production %acility layouts.
Answer: 0n a custom1products layout" e7uipment and people are grouped according to %unctionE
in a same1steps layout" e7uipment and people are set up to produce one type o% product in a %ied
se7uence o% steps and are arranged according to production re7uirements.
Eplanation: A custom1products layout is well suited %or make1to1order shops while a same1
steps layout is e%%icient %or make1to1stock operations.
$age Re%: 1*511*8
'i%%iculty: 'i%%icult
(b)ecti#e: *.5
+earning (utcome: 'iscuss the %actors that in%luence decisions about organi,ational structure
-kill: .oncept
3&
.opyright 9 /213 $earson Education" 0nc. $ublishing as $rentice :all
11/) Eplain how A0T production and lean manu%acturing are related.
Answer: +ean manu%acturing is designed %or smooth production %lows that a#oid ine%%iciencies"
eliminate unnecessary in#entories" and continuously impro#e production processes. Aust1in1time
BA0T) production" a type o% lean system" brings together all needed materials at the precise
moment they are re7uired %or each production stage.
Eplanation: ?ith A0T production" disruptions are more #isible and are resol#ed more 7uickly.
$age Re%: 1<2
'i%%iculty: 'i%%icult
(b)ecti#e: *.&
+earning (utcome: 'iscuss the %actors that in%luence decisions about organi,ational structure
-kill: -ynthesis
113) 0n a T;> en#ironment" what are some steps that companies use to emphasi,e the
importance o% 7ualityG
Answer: To ensure high17uality goods and ser#ices" many %irms assign responsibility %or some
aspects o% T;> to speci%ic departments or positions. >ore broadly" leaders o% the 7uality
mo#ement use #arious methods and resources to %oster a 7uality %ocus@training" #erbal
encouragement" teamwork" and tying compensation to work 7uality.
Eplanation: Total 7uality management BT;>) includes all the acti#ities necessary %or getting
high17uality goods and ser#ices into the marketplace.
$age Re%: 1<3
'i%%iculty: 'i%%icult
(b)ecti#e: *.*
+earning (utcome: 'escribe the skills and %unctions o% management
-kill: .oncept
115) ?hat is a supply chainG :ow can supply chain management create competiti#e ad#antageG
Answer: A supply chain is the %low o% in%ormation" materials" and ser#ices that starts with raw
materials suppliers and continues through other steps in the operations process until the product
reaches the end consumer. -upply chain management creates better #alue %or the customer to
produce a competiti#e ad#antage %or the members o% the chain.
Eplanation: 'ell" 0nc. has used an inno#ati#e supply chain strategy to help lower prices and
speed deli#ery o% its $.s.
$age Re%: 1<811<&
'i%%iculty: 'i%%icult
(b)ecti#e: *.<
+earning (utcome: 'escribe the skills and %unctions o% management
-kill: .oncept
3*
.opyright 9 /213 $earson Education" 0nc. $ublishing as $rentice :all
118) :ow does outsourcing a%%ect global supply chain managementG
Answer: (utsourcing is the strategy o% paying suppliers and distributors to per%orm certain
business processes or to pro#ide needed materials or ser#ices. Arrangements %or cross1border
materials %lows re7uire compliance with each country4s commerce regulations. $roduction and
global transportation scheduling are coordinated with U.-. market demand so that outsourced
products arri#e in the correct amount and on time without harming the manu%acturer4s image.
Eplanation: (utsourcing has been a growing trend in American business. (utsourcers ha#e a
greater need o% operations skills %or integration among dispersed %acilities.
$age Re%: 1<&11<*
'i%%iculty: 'i%%icult
AA.-6: 'ynamics o% the global economy
(b)ecti#e: *.<
+earning (utcome: Eplain the bene%its and challenges o% engaging in international business
-kill: -ynthesis
11&) Eplain time" place" and %orm utility.
Answer: Time utility is created when a company makes products a#ailable when consumers
want them. Eamples may include seasonal items such as .hristmas ornaments" bathing suits"
and mos7uito repellent. $lace utility is created when a company makes a product4s location
con#enient %or consumers. An eample is the placement o% routinely purchased items in
con#enience stores or con#eniently located discount stores. Form utility is created when a
company combines materials to create products.
Eplanation: A business adds customer #alue by pro#iding utility.
$age Re%: 1&*
'i%%iculty: >oderate
(b)ecti#e: *./
+earning (utcome: 'iscuss the roles o% ethics and corporate responsibility in business
-kill: Application
11*) The tet says that Toyota has %ocused on 7uality as its operations capability. Eplain how
Toyota4s operations might ha#e changed i% it had %ocused on low cost instead o% 7uality.
Answer: A cost1minimi,ation %ocus would ha#e been appropriate in this case" gi#ing Toyota4s
operations an altogether di%%erent %orm. $ossible changes include more outsourcing o%
production" reduced ad#ertising" %ewer models" and %ewer accessories on the cars.
Eplanation: (perations capability is the particular ability that production does especially well
to outper%orm the competition.
$age Re%: 1*1
'i%%iculty: 'i%%icult
AA.-6: Analytic skills
(b)ecti#e: *.3
+earning (utcome: 'iscuss the %actors that in%luence decisions about organi,ational structure
-kill: Application
3<
.opyright 9 /213 $earson Education" 0nc. $ublishing as $rentice :all
11<) 'escribe the %i#e most commonly used tools %or T;>.
Answer: The most commonly used tools %or T;> are #alued1added analysis" 7uality
impro#ement teams" getting closer to the customer" the 0-( series" and business process
reengineering. Falue1added analysis re%ers to the e#aluation o% all work acti#ities" material %lows"
and paperwork to determine the #alue that they add %or customers. ;uality impro#ement teams
are groups o% employees %rom #arious work areas who meet regularly to de%ine" analy,e" and
sol#e common production problems. !etting closer to the customer in#ol#es taking steps to
know what customers want in the products they consume. The 0-( series pertains to a series o%
certi%ications attesting that a %actory" laboratory" or o%%ice adheres to rigorous 7uality
management re7uirements set by the 0nternational (rgani,ation %or -tandardi,ation. 6usiness
process reengineering %ocuses on impro#ing a business process@rethinking each o% the process4s
steps by starting %rom scratchE the process yields impro#ements as measured by cost" 7uality"
speed" and ser#ice.
Eplanation: T;> begins with leadership and a desire %or continuously impro#ing both
processes and products. 0t must consider all aspects o% a business" including customers" suppliers"
and employees.
$age Re%: 1<311<8
'i%%iculty: 'i%%icult
(b)ecti#e: *.*
+earning (utcome: 'escribe the skills and %unctions o% management
-kill: .oncept
11=) :ow are !eneral >otors and .hrysler responding to current economic conditionsG
Answer: 6oth companies ha#e adopted leaner operations to simpli%y production.
Eplanation: 0n a slow economy" businesses maintain pro%itability through cutting costs and
impro#ing e%%iciency.
$age Re%: 1</
'i%%iculty: >oderate
(b)ecti#e: *.&
+earning (utcome: 'iscuss the %actors that in%luence decisions about organi,ational structure
-kill: .oncept
1/2) :ow ha#e !eneral >otors and .hrysler implemented leaner operationsG
Answer: A smaller number o% makes" models" and options simpli%ies product design" production"
and distribution.
Eplanation: !>" .hrysler" and Ford are adopting business strategies that Aapanese producers
ha#e used to simpli%y production and capture a greater market share.
$age Re%: 1</
'i%%iculty: >oderate
(b)ecti#e: *.&
+earning (utcome: 'iscuss the %actors that in%luence decisions about organi,ational structure
-kill: Application
3=
.opyright 9 /213 $earson Education" 0nc. $ublishing as $rentice :all
1/1) ?hat changes can be epected in !eneral >otors and .hrysler4s distribution networks as a
result o% leaner operationsG
Answer: The companies4 distribution networks will be simpli%ied as some brands are eliminated.
Fewer brands will reduce the number o% auto dealerships and lower distribution costs.
Eplanation: !> has downsi,ed to %our core brands and .hrysler4s roster includes only three
ma)or brands.
$age Re%: 1</
'i%%iculty: 'i%%icult
(b)ecti#e: *.&
+earning (utcome: 'escribe the ma)or components o% e%%ecti#e distribution
-kill: -ynthesis
1//) 'escribe how a !antt chart and a $ERT chart are similar" and how they are di%%erent.
Answer: 6oth are scheduling tools used in pro)ect management" and both break down a pro)ect
into the steps to be per%ormed and show the time needed %or each step. The $ERT chart shows
the necessary se7uence o% acti#ities and identi%ies the critical path" which is the most time1
consuming set o% acti#ities. Thus the $ERT chart pro#ides e#en more in%ormation than the !antt
chart.
Eplanation: $ro)ect managers may need to reassign workers and e7uipment to speed up late
acti#ities and stay on schedule.
$age Re%: 1*<
'i%%iculty: 'i%%icult
AA.-6: Analytic skills
(b)ecti#e: *.8
+earning (utcome: 'iscuss the %actors that in%luence decisions about organi,ational structure
-kill: .oncept
1/3) FedE and the U.-. $ostal -er#ice compete %or many o% the same customers. :ow are their
strategies %or attracting customers di%%erentG
Answer: FedE emphasi,es dependability and a speci%ic deli#ery time. The U.-. $ostal -er#ice
doesn4t speci%y a deli#ery date %or %irst1class mail" but U.-. mail is less epensi#e. FedE tends to
%ocus more on business customers than does the postal ser#ice. The postal ser#ice is more
accessible" with daily deli#eries to homes. 0n general" the postal ser#ice pursues a lower1cost
strategy.
Eplanation: ?ith products such as priority mail and epress mail" the postal ser#ice is
increasingly emphasi,ing dependable deli#ery.
$age Re%: 1*1
'i%%iculty: 'i%%icult
AA.-6: Analytic skills
(b)ecti#e: *.3
+earning (utcome: 'iscuss the %actors that in%luence decisions about organi,ational structure
-kill: .oncept
52
.opyright 9 /213 $earson Education" 0nc. $ublishing as $rentice :all
1/5) 'escribe three ways that >c'onald4s restaurants create time utility %or customers.
Answer: >c'onald4s ser#es its %ood 7uickly" according to customer wishes. >any outlets ser#e
break%ast during the morning hours. 'ri#e1through windows allow customers to order without
ha#ing to park and enter the restaurant. >any outlets are open late to ser#e customers.
Eplanation: A business creates time utility when it pro#ides products when consumers want
them.
$age Re%: 1&*
'i%%iculty: 'i%%icult
(b)ecti#e: *./
+earning (utcome: 'iscuss the %actors that in%luence decisions about organi,ational structure
-kill: .oncept
1/8) -uppose that economists are %orecasting a short economic recession %ollowed by strong
economic growth. :ow might a large manu%acturing company ad)ust its capacity planning to
respond to this %orecastG
Answer: The company will probably wish to reduce its capacity in the short run" but retain its
ability to increase capacity when the economy strengthens. .ompany eecuti#es may decide to
reduce worker hours temporarily instead o% laying o%% workers. They may decide to idle some
%acilities but keep them a#ailable to resume operations when economic growth picks up.
Eplanation: A %irm4s capacity depends on how many people it employs and the number and si,e
o% its %acilities.
$age Re%: 1*5
'i%%iculty: 'i%%icult
AA.-6: Analytic skills
(b)ecti#e: *.5
+earning (utcome: 'escribe the skills and %unctions o% management
-kill: -ynthesis
1/&) 0n recent years" some automakers ha#e increased manu%acturing operations in the
southeastern United -tates. -uggest two reasons why they ha#e taken this step.
Answer: The cost o% li#ing tends to be lower in this area" allowing companies to o%%er lower
wages. There is also less union presence than in most other areas o% the country. -ome state
go#ernments ha#e adopted policies" such as ta breaks" to encourage manu%acturing in their
states.
Eplanation: 6ecause location a%%ects production costs and %leibility" sound location planning is
crucial %or %actories" o%%ices" and stores.
$age Re%: 1*5
'i%%iculty: 'i%%icult
AA.-6: Analytic skills
(b)ecti#e: *.5
+earning (utcome: 'iscuss the %actors that in%luence decisions about organi,ational structure
-kill: Application
51
.opyright 9 /213 $earson Education" 0nc. $ublishing as $rentice :all
1/*) (utsourcing has led to increasingly global supply chains. 'escribe three ways that a more
global supply chain might be risky %or a company that outsources.
Answer: ?hile outsourcing has usually been pro%itable" it has some risks. !lobal supply chains
are more #ulnerable to natural disasters" such as the earth7uake and tsunami in Aapan and
%looding in Thailand. $olitical unrest in other countries is also a risk. -ome countries ha#e
nationali,ed businesses without warning to the companies. 0t is possible that U.-. lawmakers
could impose laws discouraging outsourcing. .riminal acti#ities and e#en war could also disrupt
supply chains.
Eplanation: (utsourcing epands supply chains and increases their compleity.
$age Re%: 1<&11<*
'i%%iculty: 'i%%icult
AA.-6: 'ynamics o% the global economy
(b)ecti#e: *.<
+earning (utcome: Eplain the bene%its and challenges o% engaging in international business
-kill: .oncept
Alan Aackson is the %ounder and owner o% -ky -cooters" a small manu%acturing company located
in -an Aose" .ali%ornia. For years" Alan toyed with ideas %or a motori,ed scooter. -eeing the
success o% the -egway scooter" Alan decided to act. Although his design" the -ky .ruiser" is not
as well1known as the -egway" it incorporates many o% the same bene%its. Adding to its appeal"
the -ky .ruiser sells %or almost J1"222 less than the -egway. Firtually all -ky .ruisers are sold
through direct appeals on the ;F. shopping network. (#erwhelmed by recent demand" Alan has
hired his sister" .olette. .olette" a recent business school graduate" has suggested that Alan
consider a )ust1in1time production system. Alan wonders i% this would be in his best interest.
1/<) ?ould a make1to1stock or make1to1order operations process be most e%%icient %or the
production o% -ky .ruisersG Eplain.
Answer: A make1to1stock operation would allow -ky -cooters to produce scooters with least
epense and make them a#ailable %or immediate shipment.
Eplanation: >ake1to1order operations are generally more time1consuming and more epensi#e.
$age Re%: 1&=
'i%%iculty: >oderate
AA.-6: Analytic skills
(b)ecti#e: *./
+earning (utcome: 'iscuss the %actors that in%luence decisions about organi,ational structure
-kill: Application
5/
.opyright 9 /213 $earson Education" 0nc. $ublishing as $rentice :all
1/=) ?hat ad#antages could -ky -cooters epect %rom switching to a )ust1in1time production
systemG
Answer: Aust1in1time systems sa#e money by reducing in#entories and replacing stop1and1go
production with smooth mo#ement" making disruptions more #isible. Finding and eliminating
disruptions by continuous impro#ement o% production is a ma)or ob)ecti#e o% A0T.
Eplanation: Aust1in1time production is an eample o% a lean production system.
$age Re%: 1<2
'i%%iculty: >oderate
AA.-6: Analytic skills
(b)ecti#e: *.&
+earning (utcome: 'iscuss the %actors that in%luence decisions about organi,ational structure
-kill: Application
132) Are there any potential negati#e conse7uences o% switching to a )ust1in1time production
systemG Eplain.
Answer: A0T production could ha#e negati#e conse7uences i% not well planned. -ince the
re7uired materials arri#e )ust as they are needed" poor planning could shut down the whole
production line. -imilarly" %ailure by any o% the members o% the supply chain could erase the
bene%it o% the A0T strategy. Alan should care%ully plan the implementation o% a A0T production
system.
Eplanation: A0T production reduces costs when implemented properly" but it re7uires more
o#ersight than a traditional production system.
$age Re%: 1<2
'i%%iculty: 'i%%icult
AA.-6: Analytic skills
(b)ecti#e: *.&
+earning (utcome: 'iscuss the %actors that in%luence decisions about organi,ational structure
-kill: Application
131) !i#en the price o% the -ky .ruiser" what is a reasonable goal %or product 7ualityG
Answer: The -ky .ruiser is priced about J1"222 less than its competitor" -egway. The -ky
.ruiser should be sa%e and per%orm e%%iciently" but it need not eceed the per%ormance o%
-egway. A reasonable goal is to per%orm close to the -egway standard. The -ky .ruiser will
probably ha#e %ew CetrasC and its appearance will likely be %unctional rather than sleek.
Eplanation: (%%ering a lower1cost product is one strategy that -ky -cooters will use to attract
customers.
$age Re%: 1<1
'i%%iculty: 'i%%icult
AA.-6: Analytic skills
(b)ecti#e: *.*
+earning (utcome: 'escribe the skills and %unctions o% management
-kill: Application
53
.opyright 9 /213 $earson Education" 0nc. $ublishing as $rentice :all
13/) ?hat are three possible changes that Alan Aackson can make in his company to emphasi,e
7uality impro#ementG
Answer: $ossible approaches include %orming a 7uality management team" encouraging 7uality
ownership" adopting total 7uality management" and creating a team to recei#e and re#iew input
%rom customers and potential customers.
Eplanation: 0n addition to controlling 7uality" businesses seek to build 7uality into goods and
ser#ices.
$age Re%: 1<311<5
'i%%iculty: 'i%%icult
AA.-6: Re%lecti#e thinking skills
(b)ecti#e: *.*
+earning (utcome: 'escribe the skills and %unctions o% management
-kill: Application
133) 0% a A0T production system is implemented at -ky -cooters" how might the %irm4s supply
chain strategy be a%%ectedG
Answer: Alan and .olette must coordinate -ky -cooters4 acti#ities with a network o% other
%irms. They will need to regard these %irms as members o% a coordinated supply system. A A0T
production system will be challenging since timing will be o% utmost importance. -ky -cooters
will need to communicate 7uickly and e%%ecti#ely with its partners and may need the latest
technologies in order to implement the more challenging A0T system.
Eplanation: -upply chain strategy is based on the idea that members o% the chain will gain
competiti#e ad#antage by working as a coordinated unit. Although each company looks out %or
its own interests" it works closely with suppliers and customers throughout the chain.
$age Re%: 1<2" 1<&
'i%%iculty: 'i%%icult
AA.-6: Analytic skills
(b)ecti#e: *.<
+earning (utcome: 'iscuss the %actors that in%luence decisions about organi,ational structure
-kill: Application
55
.opyright 9 /213 $earson Education" 0nc. $ublishing as $rentice :all
EKTrade is a retail organi,ation that owns specialty stores %or a di#erse range o% product
categories" ranging %rom home %urnishings and sporting goods to books and gourmet %oods. ?ith
more than 82 stores" the %irm is now considering entering the #ery lucrati#e low1cost market by
launching .lotheLa chain o% retail stores that o%%ers a%%ordable %ashion %or men" women" and
children.
135) ?hich o% the %ollowing" i% true" would strengthen the argument %or being a low1cost playerG
A) .ustomers who belie#e that they need high le#els o% customer ser#ice are more likely to
choose a niche retailer.
6) An epected upturn in the economy will increase consumer purchasing power.
.) 6eing the low1cost player re7uires a company to eamine e#ery aspect o% its supply chain in
order to identi%y opportunities to impro#e e%%iciency.
') EKTrade has a longer history selling clothes %or men and women than it has selling clothes %or
children.
E) The retailer4s logistics operations are more e%%icient than the industry norm.
Answer: E
Eplanation: E) .ompeting on price isn4t easy. Hot e#ery organi,ation can do it. :owe#er"
organi,ations that are already e%%icient are better suited to try to win as a low1cost player. -o
.hoice E strengthens the argument. .hoices A" 6" and . weaken the argument by suggesting
problems with the low1cost approach. .hoice ': The length o% acti#ity in di%%erent product lines
doesn4t help resol#e the 7uestion o% whether low1cost is the way to go.
$age Re%: 1&&
'i%%iculty: >oderate
AA.-6: Re%lecti#e thinking skills
(b)ecti#e: *.1
-kill: .ritical Thinking
58
.opyright 9 /213 $earson Education" 0nc. $ublishing as $rentice :all
-tardust 4n4 .lay is a manu%acturer and mass marketer o% personal care products and nutritional
supplements. The company plans to launch a line o% organic beauty products that are made
entirely %rom mineral pigments and organic plant etracts. These products" though epensi#e to
process" o%%er much higher margins than the company4s current lines o% beauty products. The
market %or organic beauty products is relati#ely untapped" with )ust a hand%ul o% players.
138) ?hich o% the %ollowing" i% true" would strengthen the case %or -tardust 4n4 .lay to begin
de#eloping organic beauty productsG
A) Retailers report a consumer pre%erence %or inepensi#e skin1care and beauty products that do
a better )ob than the ones currently a#ailable.
6) (rganic raw materials sourced %rom economically weak countries pro#ide a source o%
li#elihood %or the local population.
.) (rganic raw materials %or such products can be sourced %rom de#eloping countries at a lower
cost.
') The %irst mo#er in this industry is witnessing reasonable growth.
E) A growing number o% women 1<135 consider the health impact o% chemicals %ound in a beauty
product when making a cosmetics purchase decision.
Answer: E
Eplanation: E) -tardust 4n4 .lay sees many potential ad#antages in marketing organic beauty
products. Are there enough people interested in the product to make the higher production costs
worthwhileG .hoice E suggests there are" indicating that young women@who comprise a large
cosmetics market@care about the chemicals in their beauty products. .hoice A does not say
much about organic products. Also" organic products are epensi#e to manu%acture and o%%er
higher margins. 0n other words" they are more epensi#e than nonorganic beauty products.
.hoice 6 eplains how a new -tardust 4n4 .lay line o% organic beauty products might bene%it
people in economically weak countries" but it tells us nothing about whether the line would
bene%it -tardust 4n4 .lay. .hoice . pro#ides general in%ormation about raw material sourcing %or
the proposed line. This is not an absolute ad#antage %or -tardust 4n4 .lay" as the same conditions
apply to competitors. .hoice ' only tells us that the %irst mo#er in the market has seen
Creasonable growth.C This %irst mo#er might #ery well be a small niche marketer. -tardust 4n4
.lay is a mass marketer" so Creasonable growthC to a smaller company may be insigni%icant to
-tardust 4n4 .lay.
$age Re%: 1*2
'i%%iculty: >oderate
AA.-6: Re%lecti#e thinking skills
(b)ecti#e: *.3
-kill: .ritical Thinking
5&
.opyright 9 /213 $earson Education" 0nc. $ublishing as $rentice :all
13&) ?hich o% the %ollowing" i% true" would weaken the argument %or de#eloping organic beauty
productsG
A) Few companies ha#e obtained patents %or their organic product inno#ations.
6) .ompetitors in this segment are 7uick to copy any new product inno#ation and mimic product
design.
.) (rganic o%%erings ha#e a shorter shel%1li%e compared to synthetic products.
') .ustomers need to be educated about the relati#e bene%its o%%ered by organic products.
E) Financial analysis shows that it would take the company 1.8 years to reco#er its initial
in#estment.
Answer: 6
Eplanation: 6) 0% -tardust doubts whether it can achie#e and maintain a competiti#e ad#antage
in the organic beauty products market" then the company is unlikely to pursue the proposed line.
.hoice 6 points to a lack o% sustainable competiti#e ad#antage" as competitors can be epected
to 7uickly copy -tardust4s product and promotions. .hoice 6 weakens the argument. .hoice A is
a nonissue" as -tardust could use di%%erent processes and come up with di%%erent products. .hoice
. doesn4t necessarily weaken the argument" as a shorter shel%1li%e suggests that consumers would
need to repurchase the cosmetics more %re7uently. .hoice ' indicates a need %or increased
customer awareness" a promotional challenge that would be true %or many inno#ati#e products.
Alone" this isn4t enough. .hoice E tells us how long it would take the company to reco#er its
in#estment" but without a %rame o% re%erence we don4t know i% this strengthens or weakens the
argument. 0% it usually takes two or more years to reco#er an in#estment" then this strengthens the
argument %or de#eloping an organic line.
$age Re%: 1*2
'i%%iculty: >oderate
AA.-6: Re%lecti#e thinking skills
(b)ecti#e: *.3
-kill: .ritical Thinking
5*
.opyright 9 /213 $earson Education" 0nc. $ublishing as $rentice :all
Toys %or $ops is a small business that %ocuses on indoor and outdoor games intended %or an adult
male clientele. 0t stocks a range o% %antasy action %igures and models" low1impact athletic
accessories such as Frisbees and cro7uet sets" and a wide range o% #ideo games. The business has
grown to the point that a larger location is needed. Recently" a location has come up %or rent at a
nearby strip mall at a %a#orable rate. The %acility would pro#ide the business with much needed
shel% space and more room %or employees behind the counter. :owe#er" the strip mall is in an
area that has also seen a rapid increase in petty crime. The .E( %eels that the ad#antages o% the
new location support the decision to relocate.
13*) ?hich o% the %ollowing" i% true" would weaken the .E(4s positionG
A) The i%%y reputation o% the area in which the strip mall is located would make the business4s
current employees uneasy about their sa%ety at work" which would lead to increased turno#er in
sta%%.
6) The lower cost o% li#ing in the new neighborhood would make it easier to hire employees
willing to accept lower salaries.
.) Fideo games sales" a%ter se#eral years o% increases" ha#e reached a plateau and are unlikely to
show %urther growth in the net %ew years.
') The strip mall has no other current #acancies.
E) The current sta%% has been stable %or se#eral years and is well #ersed in details o% #ideo games
and other specialty items.
Answer: A
Eplanation: A) Two o% the key human resource management responsibilities o% a small business
are maintaining morale and protecting employees4 physical conditions. ?hen these issues are
neglected" turno#er increases" leading to the need to hire and train new sta%% more o%ten than
necessary. :ence .hoice A weakens the .E(4s position. .on#ersely" .hoice 6 might strengthen
the decision" by showing that labor costs can be controlled through the mo#e. .hoice . might be
a consideration i% the business sold only #ideo games" but in %act this is )ust one product line the
%irm handles. .hoice '" i% anything" strengthens the .E(4s position by suggesting that the strip
mall is being e%%ecti#ely managed by its owners. .hoice E deals with a strength o% the company
in its eisting location" but does not address the wisdom o% the mo#e.
$age Re%: 1*5
'i%%iculty: >oderate
AA.-6: Re%lecti#e thinking skills
(b)ecti#e: *.5
-kill: .ritical Thinking
5<
.opyright 9 /213 $earson Education" 0nc. $ublishing as $rentice :all
13<) ?hich o% the %ollowing" i% true" might )usti%y the relocation as a prudent mo#e %rom the
human resource point o% #iewG
A) 'e#eloping a Facebook page made the business known to a large group o% potential
customers li#ing in a much wider regional area.
6) Federal crime reports show that the greater metro area has seen an increase in car the%ts.
.) The new location is located near a ma)or %reeway interchange.
') Fantasy action %igures are o%ten remaindered %or below wholesale cost because their
popularity is short1li#ed.
E) Anonymous notes in the %irm4s suggestion bo repeatedly cited cramped conditions and lack
o% pri#acy as the most annoying %actors in working conditions at the present site.
Answer: E
Eplanation: E) ?hile the potential %or crime is troublesome" employees might consider the
trade1o%% acceptable i% the mo#e results in a signi%icant impro#ement in working conditions. -o
.hoice E is the strongest argument in %a#or o% the mo#e. .hoice A doesn4t address the human
resource impact o% relocating to a marginal neighborhood. .hoices 6 and ' ha#e no rele#ance to
the proposed mo#e. .hoice . might support the mo#e by suggesting that the location would be
an easier commute %or employees" but it4s also possible that the interchange area would be a
tra%%ic bottleneck.
$age Re%: 1*5
'i%%iculty: >oderate
AA.-6: Re%lecti#e thinking skills
(b)ecti#e: *.5
-kill: .ritical Thinking
5=
.opyright 9 /213 $earson Education" 0nc. $ublishing as $rentice :all
.lean -weep .orporation is a consumer goods company manu%acturing home1care cleaning
products. -ales o% -parkle" its %lagship disin%ectant" and some o% its other cleaning products ha#e
been sluggish during the last two 7uarters. (% late" retailers ha#e also shown a lack o% con%idence
in these product lines. The management is looking at di%%erent ways to impro#e the e%%iciency o%
its entire supply chain. -ome senior eecuti#es belie#e that the company should shi%t to a )ust1in1
time logistics system. :owe#er" this opinion does not ha#e unanimous support.
13=) ?hich o% the %ollowing" i% true" would weaken the argument %or the shi%t to a )ust1in1time
logistics systemG
A) .arrying too much in#entory results in higher1than1necessary in#entory1carrying costs.
6) Under the new system" the company would ha#e in#entory su%%icient %or %ewer days o%
operations.
.) .lean -weep4s most important suppliers pre%er a %leible supply schedule.
') 0mplementing the system would re7uire an etensi#e o#erhaul o% the eisting supply chain.
E) 'emand %or .lean -weep4s products is stable and predictable.
Answer: '
Eplanation: ') .hoice ' weakens the argument since the bene%its o% a )ust1in1time system
could be outweighed by the costs o% re1engineering the entire supply chain. .hoice A: A )ust1in1
time system is designed to lower in#entory1carrying costs" so .hoice A would strengthen the
argument. .hoice 6 describes the system accurately but pro#ides no reason not to adopt it.
.hoice . would strengthen the argument by suggesting that the transition could be easy. .hoice
E strengthens the argument by suggesting that it will be possible to operate with lower in#entory
le#els.
$age Re%: 1<2
'i%%iculty: >oderate
AA.-6: Re%lecti#e thinking skills
(b)ecti#e: *.&
-kill: .ritical Thinking
82
.opyright 9 /213 $earson Education" 0nc. $ublishing as $rentice :all
152) ?hich o% the %ollowing" i% true" would strengthen the argument %or the shi%t to a )ust1in1time
in#entory systemG
A) .lean -weep4s storage and handling costs are signi%icant.
6) .lean -weep4s suppliers ha#e not adopted )ust1in1time systems.
.) The current system re7uires higher in#entory le#els during the summer as compared to the
winter.
') 'emand %or -parkle #aries depending on the promotional strategies o% .lean -weep4s retail
partners.
E) -parkle comes in many #arieties" none o% which is more popular than the leading brand.
Answer: A
Eplanation: A) Aust1in1time logistics systems can reduce handling and storage costs" which is
an ad#antage when those costs are signi%icant" .hoice A. .hoice 6" i% anything" weakens the
argument" as the system might be e#en more e%%ecti#e i% others in the supply chain used similar
systems. .hoice . does not point out a %law in the current system" as it is reasonable that people
would use more cleaning products in warmer weather. .hoice ' suggests that demand is hard to
predict" which would weaken the argument. .hoice E: -parkle4s position in the market does not
tell which in#entory system makes the most sense.
$age Re%: 1<2
'i%%iculty: >oderate
AA.-6: Re%lecti#e thinking skills
(b)ecti#e: *.&
-kill: .ritical Thinking
151) ?hich o% the %ollowing 7uestions is +EA-T rele#ant to the issue o% whether the company
should switch to a )ust1in1time logistics systemG
A) 0s demand %or .lean -weep4s products predictableG
6) .an suppliers assure %ast" %re7uent" and %leible deli#eryG
.) 0s technology a#ailable to manage the logisticsG
') 'o end users currently %a#or making the transitionG
E) 'oes .lean -weep ha#e access to accurate in%ormation about customer demandG
Answer: '
Eplanation: ') All the choices speak to re7uirement %or a )ust1in1time system ecept .hoice '.
A success%ul )ust1in1time system would be in#isible to most customers. They would simply ha#e
access to the product when they wanted it. Also" customers who buy cleaning supplies would not
ha#e an in%ormed opinion o% the company4s supply chain operation" so their initial opinion is not
crucial here.
$age Re%: 1<2
'i%%iculty: 'i%%icult
AA.-6: Re%lecti#e thinking skills
(b)ecti#e: *.&
-kill: .ritical Thinking
81
.opyright 9 /213 $earson Education" 0nc. $ublishing as $rentice :all
15/) A senior eecuti#e claims that a mo#e to a C)ust1in1timeC logistics system would be a
mistake because e#en i% the system is e%%ecti#e at reducing in#entory and handling costs" the
change would not be noticed by customers. The eecuti#e claims that the change should be
re)ected as it runs counter to .lean -weep4s goal o% being Ccustomer1%ocused.C ?hich o% the
%ollowing is the best criticism o% the senior eecuti#e4s positionG
A) 0t does not pro#e that %ocusing on the customer helps an organi,ation become success%ul.
6) 0t does not suggest an impro#ement to .lean -weep4s current system.
.) 0t %ails to recogni,e that ha#ing a more e%%icient supply chain will allow .lean -weep to o%%er
more #alue to its customers.
') 0t does not establish that the costs o% switching to the new system would outweigh any cost1
reduction bene%its the system might pro#ide.
E) 0t does not describe the in#entory systems used by .lean -weep4s competitors.
Answer: .
Eplanation: .) The senior eecuti#e is basically saying that any change would ha#e to be
noticed by customers in order %or the change to be a good one. 6ut that doesn4t make sense. All
other things being e7ual" sa#ing money in production and distribution is a good thing" e#en i% the
customers don4t notice the changes. 6esides" as .hoice . points out" ha#ing lower costs can
allow .lean -weep to o%%er more #alue" %or eample by lowering prices. .hoice A: The #alue o%
being customer1%ocused isn4t at issue here. The problem is that the senior eecuti#e takes it too
%ar in claiming that e#ery good idea must be percei#ed by the customer. .hoices 6" '" and E are
accurate in that the argument does not do these things" but there is no reason to belie#e that they
are necessary.
$age Re%: 1<2
'i%%iculty: 'i%%icult
AA.-6: Re%lecti#e thinking skills
(b)ecti#e: *.&
-kill: .ritical Thinking
8/
.opyright 9 /213 $earson Education" 0nc. $ublishing as $rentice :all
A so%tware de#elopment company is designing an e#aluation plan %or its so%tware programmers.
The company %eels that changes are necessary because it lacks the %acts it needs to distinguish
outstanding so%tware programmers %rom those that are only a#erage" or worse. $re#iously" the
company paid so%tware programmers a %lat salary and based e#aluations on super#isors4
opinions. How" howe#er" the company is considering the %ollowing measures %or its so%tware
programmers:
>easurement -trategy Alpha: -o%tware programmers will be e#aluated based on the total
number o% lines o% code that they produce.
>easurement -trategy 6eta: -o%tware programmers will be e#aluated based on their ability to
produce computer code that is %ree o% errors.
>easurement -trategy !amma: -o%tware programmers will be e#aluated based on the market
success o% the products they produce.
153) ?hich o% the %ollowing" i% true" would weaken the argument that the company should use
>easurement -trategy AlphaG
A) -o%tware programmers usually ha#e no direct contact with customers and do not determine
product speci%ications.
6) The programming languages used by the company re7uire %ewer lines o% code to per%orm a
%unction than the programming languages used by other companies.
.) Any so%tware programmer can choose to increase the number o% lines o% code it takes to
per%orm any %unction.
') An e%%ecti#e so%tware programming team is essential %or the %inancial success o% the company.
E) >any o% the current so%tware programmers would not recei#e strong e#aluations i% the
company used >easurement -trategy Alpha.
Answer: .
Eplanation: .) >easurement -trategy Alpha rewards so%tware programmers based on the
number o% lines o% code that they produce. 6ut i% .hoice . is true then the so%tware programmers
can arti%icially in%late the number o% lines it takes to do something. They could abuse this
approach by taking thousands o% lines o% code to do simple things. They would seem producti#e
according to >easurement -trategy Alpha" but they really wouldn4t be. .hoice A is sort o%
rele#ant to >easurement -trategy !amma" not Alpha. .hoice 6 is a reason why >easurement
-trategy Alpha couldn4t be used to compare programmers %rom di%%erent companies" but these
strategies concern the company4s employees only. .hoice ' suggests that getting this right is
important but does not tell us whether Alpha will help the company get it right. .hoice E would
be a problem i% we knew that all o% the current so%tware programmers are ecellent" but the
whole point o% the measurement plan is that we don4t know who is doing a good )ob. -o the %act
that Alpha would be bad %or some employees doesn4t suggest that there is anything wrong with
Alpha.
$age Re%: 1<2
'i%%iculty: >oderate
AA.-6: Re%lecti#e thinking skills
(b)ecti#e: *.&
-kill: .ritical Thinking
83
.opyright 9 /213 $earson Education" 0nc. $ublishing as $rentice :all
155) A decision to use >easurement -trategy 6eta would assume which o% the %ollowingG
A) -o%tware errors are more common than hardware errors.
6) -o%tware errors a%%ect all users e7ually.
.) The impact o% so%tware errors has been greater recently.
') ?hen a so%tware error occurs" its e%%ects are always clear to the %inal user o% the product.
E) ?hen a so%tware error is disco#ered" it is possible to determine who is responsible %or it.
Answer: E
Eplanation: E) >easurement -trategy 6eta punishes people %or their errors" but i% .hoice E
isn4t true" then you can4t tell who made the error. .hoice E would ha#e to be true in order %or
>easurement -trategy 6eta to work" and so it must be assumed. .hoice A: This is about
so%tware" not hardware" so nothing about hardware errors need be assumed. .hoice 6 could be
true" and probably isn4t" but it doesn4t need to be true in order %or 6eta to work. .hoices . and '
would strengthen the case %or 6eta i% they were true" but they don4t need to be true in order %or
6eta to work.
$age Re%: 1<2
'i%%iculty: 'i%%icult
AA.-6: Re%lecti#e thinking skills
(b)ecti#e: *.&
-kill: .ritical Thinking
158) The choice to use only >easurement -trategy 6eta would be #ulnerable to criticism
because it would gi#e the so%tware programmers an incenti#e to do whatG
A) make o#erly optimistic commitments to customer1%acing employees at the company
6) spend too much time checking their code in order to eliminate any possibility o% error
.) de#elop pro)ects that meet programming re7uirements but not market re7uirements
') skip the 7uality assurance checks that re#eal the most damaging so%tware errors
E) produce more lines o% code than is re7uired to per%orm a gi#en %unction
Answer: 6
Eplanation: 6) A#oiding errors is good" but so is %inishing on time. 0% the %ocus is only on
a#oiding errors" then the so%tware programmers will spend way too much time a#oiding any
chance that there is an error in their code somewhere. .hoice 6 describes this issue. .hoices A
and . are rele#ant to !amma. .hoice ' would be sol#ed by 6eta. .hoice E is a problem with
Alpha.
$age Re%: 1<2
'i%%iculty: >oderate
AA.-6: Re%lecti#e thinking skills
(b)ecti#e: *.&
-kill: .ritical Thinking
85
.opyright 9 /213 $earson Education" 0nc. $ublishing as $rentice :all
15&) The choice to use only >easurement -trategy !amma would be #ulnerable to criticism in
what wayG
A) >any %actors other than so%tware programming are re7uired %or market success.
6) 0t sets up a goal that is unattainable.
.) 0t underestimates the e%%ect that good so%tware programming can ha#e on the customer4s
eperience.
') 0t would re7uire the company to change its current approach to e#aluating so%tware
programmers.
E) 0t %ails to establish a goal that re7uires any signi%icant e%%ort to achie#e.
Answer: A
Eplanation: A) 0t makes sense i% the programmers are rewarded when the products do well" but
i% they are e#aluated solely on that measure" then the results might wind up being arbitrary. As
.hoice A points out" e#en i% the programmers do their )ob well" many things need to happen
be%ore the product becomes a success. (ne principle o% moti#ation is that i% you are going to
reward people when a goal is achie#ed" then those people should be able to make that goal
happen. >easurement -trategy !amma ignores this and is #ulnerable to the criticism in .hoice
A. .hoices 6 and E: There is no reason to belie#e that market success is impossible B.hoice 6)
or too easy B.hoice E). .hoice . has it backwards. 0% anything" !amma o#erestimates the
in%luence o% so%tware programming. .hoice ' is accurate but is not a problem. The whole point
is to change the system.
$age Re%: 1<2
'i%%iculty: 'i%%icult
AA.-6: Re%lecti#e thinking skills
(b)ecti#e: *.&
-kill: .ritical Thinking
15*) ?hich o% the %ollowing would be most similar to a decision to use >easurement -trategy
Alpha and >easurement -trategy 6eta togetherG
A) Aiom e#aluates its sales sta%% on the number o% sales they make and the si,e o% those sales.
6) $ersonal 'e#ices rewards its brand managers more hea#ily %or sales o% new products than %or
sales o% pre#iously eisting products.
.) -tardust 4n4 .lay rewards production line workers based on meeting production 7uotas and
a#oiding product de%ects.
') (ak >eteors customer ser#ice agents are e#aluated based on the 7uantity o% calls they answer
and the number o% %ollow1up purchases that those callers make.
E) -pa ?orld #alet parkers get a yearly bonus based on o#erall customer per%ormance and
supplement their income with tips %rom customers.
Answer: .
Eplanation: .) Alpha rewards based on output" and 6eta rewards based on a#oiding errors.
.hoice . matches this by rewarding based on 7uantity and a#oiding de%ects. Hone o% the other
choices has these components.
$age Re%: 1<2
'i%%iculty: 'i%%icult
AA.-6: Re%lecti#e thinking skills
(b)ecti#e: *.&
-kill: .ritical Thinking
88
.opyright 9 /213 $earson Education" 0nc. $ublishing as $rentice :all
15<) >anagement has decided to combine all three >easurement -trategies in order to gi#e the
so%tware programmers incenti#es that match the company4s goals. 0% the products meet their
!amma goals" so%tware programmers who ha#e met either their Alpha goals or their 6eta goals
will be rewarded" but i% the products %ail to meet their !amma goals" so%tware programmers will
not be rewarded unless they ha#e met both their Alpha goals and their 6eta goals. Under this
system" which o% the %ollowing .AHH(T be trueG
A) A so%tware programmer who has met his or her Alpha goals is not rewarded.
6) A so%tware programmer who has not met his or her 6eta goals is rewarded.
.) A so%tware programmer who has met neither his or her Alpha goals nor his or her 6eta goals
is not rewarded.
') The company meets its !amma goals" and a so%tware programmer who meets his or her
Alpha goals is not rewarded.
E) The company does not meet its !amma goals" and a so%tware programmer who meets his or
her 6eta goals is not rewarded.
Answer: '
Eplanation: ') 0% the company hits its !amma goals" then the so%tware programmers need to
hit )ust one o% Alpha and 6eta in order to be rewarded. -o .hoice ' is impossible. .hoice A
could be true i% this person did not meet his or her Alpha goals and the company did not meet its
!amma goals. .hoice 6 could be true i% this person met his or her Alpha goals and the company
met its !amma goals. .hoice .: Those who %ail both goals aren4t guaranteed a reward. .hoice E
could be true i% the person did not meet his or her Alpha goals.
$age Re%: 1<2
'i%%iculty: 'i%%icult
AA.-6: Re%lecti#e thinking skills
(b)ecti#e: *.&
-kill: .ritical Thinking
8&
.opyright 9 /213 $earson Education" 0nc. $ublishing as $rentice :all

You might also like